simplyfying ias exam preparation - insightsias · 2020. 4. 7. · insta 75 days revision plan for...

108
INSIGHTSIAS SIMPLYFYING IAS EXAM PREPARATION INSTA Tests 13 to 16 (GS) www.insightsonindia.com prelims.insightsonindia.com | mains.insightsonindia.com Telegram: insightsIAStips | FB: insightsonindia | TW: vinaygb | YT: INSIGHTS IAS BENGALURU | DELHI | HYDERABAD INSTA 75 Days REVISION PLAN UPSC Prelims 2020 Copyright © by Insights IAS All rights are reserved. No part of this document may be reproduced, stored in a retrieval system or transmitted in any form or by any means, electronic, mechanical, photocopying, recording or otherwise, without prior permission of Insights IAS. ANSWER KEY & EXPLANATIONS

Upload: others

Post on 10-Oct-2020

3 views

Category:

Documents


0 download

TRANSCRIPT

Page 1: SIMPLYFYING IAS EXAM PREPARATION - INSIGHTSIAS · 2020. 4. 7. · Insta 75 Days Revision Plan for UPSC Civil Services Prelims – 2020 This document is the compilation of 100 questions

INSIGHTSIAS SIMPLYFYING IAS EXAM PREPARATION

INSTA Tests

13 to 16 (GS)

www.insightsonindia.com

prelims.insightsonindia.com | mains.insightsonindia.com

Telegram: insightsIAStips | FB: insightsonindia | TW: vinaygb | YT: INSIGHTS IAS

BENGALURU | DELHI | HYDERABAD

INSTA 75 Days REVISION PLAN UPSC Prelims 2020

Copyright © by Insights IAS All rights are reserved. No part of this document may be reproduced, stored in a retrieval system or transmitted in any form or by any means, electronic, mechanical, photocopying, recording or otherwise, without prior permission of Insights IAS.

ANSWER KEY & EXPLANATIONS

Page 2: SIMPLYFYING IAS EXAM PREPARATION - INSIGHTSIAS · 2020. 4. 7. · Insta 75 Days Revision Plan for UPSC Civil Services Prelims – 2020 This document is the compilation of 100 questions

Insta 75 Days Revision Plan for UPSC Civil Services

Prelims – 2020

This document is the compilation of 100 questions that are part of InsightsIAS

famous INSTA REVISION initiative for UPSC civil services Preliminary examination

– 2020 (which has become most anticipated annual affair by lakhs of IAS aspirants

across the country). These questions are carefully framed so as to give aspirants tough

challenge to test their knowledge and at the same time improve skills such as

intelligent guessing, elimination, reasoning, deduction etc – which are much needed

to sail through tough Civil Services Preliminary Examination conducted by UPSC.

These questions are based on this INSTA Revision Plan which is posted on our

website (www.insightsonindia.com). Every year thousands of candidates follow our

revision timetable – which is made for SERIOUS aspirants who would like to intensively

revise everything that’s important before the exam.

Those who would like to take up more tests for even better preparation, can

enroll to Insights IAS Prelims Mock Test Series – 2020

(https://prelims.insightsonindia.com). Every year toppers solve our tests and sail

through UPSC civil services exam. Your support through purchase of our tests will help

us provide FREE content on our website seamlessly.

Wish you all the best!

Team InsightsIAS

Page 3: SIMPLYFYING IAS EXAM PREPARATION - INSIGHTSIAS · 2020. 4. 7. · Insta 75 Days Revision Plan for UPSC Civil Services Prelims – 2020 This document is the compilation of 100 questions

INSTA 75 Days REVISION PLAN for Prelims 2020 - InstaTests

www.insightsonindia.com 1 Insights IAS

DAY – 13 (InstaTest-13)

1. Consider the following statements regarding first stage of Demographic

transition

1. The first stage has high fertility and high mortality

2. The population growth is high

3. Most of the people are engaged in agriculture

Which of the statements given above is/are correct?

(a) 1 only

(b) 2 and 3 only

(c) 1 and 3 only

(d) 1, 2 and 3

Solution: C

Demographic transition

• Demographic transition theory can be used to describe and predict the

future population of any area. The theory tells us that population of any region

changes from high births and high deaths to low births and low deaths as

Page 4: SIMPLYFYING IAS EXAM PREPARATION - INSIGHTSIAS · 2020. 4. 7. · Insta 75 Days Revision Plan for UPSC Civil Services Prelims – 2020 This document is the compilation of 100 questions

INSTA 75 Days REVISION PLAN for Prelims 2020 - InstaTests

www.insightsonindia.com 2 Insights IAS

society progresses from rural agrarian and illiterate to urban industrial and

literate society. These changes occur in stages which are collectively known as

the demographic cycle.

• The first stage has high fertility and high mortality because people

reproduce more to compensate for the deaths due to epidemics and variable

food supply. The population growth is slow and most of the people are engaged

in agriculture where large families are an asset. Life expectancy is low, people

are mostly illiterate and have low levels of technology. Two hundred years ago

all the countries of the world were in this stage. Fertility remains high in the

beginning of second stage but it declines with time. This is accompanied by

reduced mortality rate. Improvements in sanitation and health conditions lead

to decline in mortality. Because of this gap the net addition to population is high.

• In the last stage, both fertility and mortality decline considerably. The

population is either stable or grows slowly. The population becomes urbanised,

literate and has high technical knowhow and deliberately controls the family

size. This shows that human beings are extremely flexible and are able to adjust

their fertility.

• In the present day, different countries are at different stages of demographic

transition.

2. Consider the following statements regarding human development

1. The concept of human development was introduced by Dr Mahbub-ul-

Haq.

2. Dr Haq has described human development as development that

enlarges people’s choices and improves their lives.

Which of the statements given above is/are correct?

(a) 1 only

(b) 2 only

(c) Both 1 and 2

(d) Neither 1 nor 2

Solution: C

• The concept of human development was introduced by Dr Mahbub-ul-Haq. Dr

Haq has described human development as development that enlarges people’s

choices and improves their lives. People are central to all development under

this concept. These choices are not fixed but keep on changing.

• The basic goal of development is to create conditions where people can live

meaningful lives. A meaningful life is not just a long one. It must be a life with

some purpose. This means that people must be healthy, be able to develop

their talents, participate in society and be free to achieve their goals.

Page 5: SIMPLYFYING IAS EXAM PREPARATION - INSIGHTSIAS · 2020. 4. 7. · Insta 75 Days Revision Plan for UPSC Civil Services Prelims – 2020 This document is the compilation of 100 questions

INSTA 75 Days REVISION PLAN for Prelims 2020 - InstaTests

www.insightsonindia.com 3 Insights IAS

3. Consider the following statements regarding Gilgit Baltistan

1. It is Located in the northern Pakistan.

2. China-Pakistan Economic Corridor (CPEC) passes through this region

3. It shares a geographical boundary with Pakistan-occupied Kashmir.

Which of the statements given above is/are correct?

(a) 1 only

(b) 1 and 2 only

(c) 2 and 3 only

(d) 1, 2 and 3

Solution: D

Home Minister Amit Shah’s statement in the Parliament has sufficiently amplified

India’s broadened vision that Pakistan Occupied Kashmir including the so-called Azad

Kashmir and Gilgit-Baltistan (G-B) belong to India.

Where is Gilgit Baltistan located?

• Located in the northern Pakistan. It borders China in the North, Afghanistan in

the west, Tajikistan in the North West and Kashmir in the south east.

• It shares a geographical boundary with Pakistan-occupied Kashmir, and India

considers it as part of the undivided Jammu and Kashmir, while Pakistan sees

it as a separate from PoK.

• It has a regional Assembly and an elected Chief Minister.

• China-Pakistan Economic Corridor (CPEC) also passes through this region.

• Gilgit-Baltistan is home to five of the “eight-thousanders” and to more than fifty

peaks above 7,000 metres (23,000 ft).

• Three of the world’s longest glaciers outside the polar regions are found in

Gilgit-Baltistan.

How Pakistan took over it?

• The British sold it, along with the rest of Jammu and Kashmir, to the Dogra ruler

of Jammu, Gulab Singh, after defeating the Sikh army in 1846, but retained

controlled over the area through a lease extracted from the Maharaja.

• This lease was last renewed in 1935. In 1947, a British army officer of the rank

of Colonel imprisoned Maharaja Hari Singh’s governor in the region, and

handed over the area for accession to Pakistan.

Recent developments:

• Pakistan, in 2017, proposed to declare the strategic Gilgit-Baltistan region as

its fifth Province.

Page 6: SIMPLYFYING IAS EXAM PREPARATION - INSIGHTSIAS · 2020. 4. 7. · Insta 75 Days Revision Plan for UPSC Civil Services Prelims – 2020 This document is the compilation of 100 questions

INSTA 75 Days REVISION PLAN for Prelims 2020 - InstaTests

www.insightsonindia.com 4 Insights IAS

Impediments ahead:

• Gilgit- Baltistan is part of J&K and any such move would seriously damage

Pakistan’s Kashmir case. Two UN resolutions of August 13, 1948 and

January 5, 1949 clearly established a link between GB and the Kashmir issue.

• Making the region its fifth province would thus violate the Karachi Agreement

— perhaps the only instrument that provides doubtful legal authority to

Pakistan’s administration of GB — as well as the UN resolutions that would

damage its position on the Kashmir issue.

• Any such move would also be violative of the 1963 Pak-China Boundary

Agreement that calls for the sovereign authority to reopen negotiations with

China “after the settlement of the Kashmir dispute between Pakistan and India”

and of the 1972 Simla Agreement that mentions that “neither side shall

unilaterally alter the situation”.

https://www.insightsonindia.com/2019/09/21/gilgit-baltistan-2/

4. Which of the following factors can be considered as Basis of International Trade

1. Difference in national resources

2. Population

3. Stage of economic development

4. Extent of foreign investment

Select the correct answer using the code given below:

(a) 1, 2 and 3 only

(b) 2, 3 and 4 only

(c) 1, 3 and 4 only

(d) 1, 2, 3 and 4

Solution: D

Basis of International Trade

(i) Difference in national resources: The world’s national resources are

unevenly distributed because of differences in their physical make up i.e.

geology, relief soil and climate.

(ii) Population factors: The size, distribution and diversity of people between

countries affect the type and volume of goods traded.

(iii) Stage of economic development: At different stages of economic

development of countries, the nature of items traded undergo changes.

(iv) Extent of foreign investment: Foreign investment can boost trade in

developing countries which lack in capital required for the development of

mining, oil drilling, heavy engineering, lumbering and plantation agriculture.

Page 7: SIMPLYFYING IAS EXAM PREPARATION - INSIGHTSIAS · 2020. 4. 7. · Insta 75 Days Revision Plan for UPSC Civil Services Prelims – 2020 This document is the compilation of 100 questions

INSTA 75 Days REVISION PLAN for Prelims 2020 - InstaTests

www.insightsonindia.com 5 Insights IAS

(v) Transport: In olden times, lack of adequate and efficient means of transport

restricted trade to local areas. Only high value items, e.g. gems, silk and spices

were traded over long distances. With expansions of rail, ocean and air

transport, better means of refrigeration and preservation, trade has experienced

spatial expansion.

5. Which of the following countries does not share its border with Saudi Arabia

(a) Iraq

(b) Iran

(c) Oman

(d) Yemen

Solution: B

Page 8: SIMPLYFYING IAS EXAM PREPARATION - INSIGHTSIAS · 2020. 4. 7. · Insta 75 Days Revision Plan for UPSC Civil Services Prelims – 2020 This document is the compilation of 100 questions

INSTA 75 Days REVISION PLAN for Prelims 2020 - InstaTests

www.insightsonindia.com 6 Insights IAS

6. Consider the following statements regarding Nuclear Suppliers Group

1. It is a multilateral export control regime.

2. India is a member of the organization.

Which of the statements given above is/are correct?

(a) 1 only

(b) 2 only

(c) Both 1 and 2

(d) Neither 1 nor 2

Solution: A

At the Bloomberg Global Business Forum in New York, Prime Minister Narendra

Modi made a pitch for India’s membership of the Nuclear Suppliers Group (NSG).

What is NSG?

• Brought in 1974– in response to the Indian nuclear test (smiling Buddha).

• It is a multilateral export control regime.

• It is a Group of nuclear supplier countries that seek to prevent nuclear

proliferation by controlling the export of materials, equipment and technology

that can be used to manufacture nuclear weapons.

• The NSG first met in November 1975 in London, and is thus popularly referred

to as the “London Club”.

• It is not a formal organization, and its guidelines are not binding. Decisions,

including on membership, are made by consensus.

Membership: 48 supplier states.

Criteria for membership:

• Ability to supply items (including items in transit) covered by the annexes to

Parts 1 and 2 of the NSG Guidelines;

• Adherence to the Guidelines and action in accordance with them;

• Enforcement of a legally based domestic export control system which gives

effect to the commitment to act in accordance with the Guidelines;

• Full compliance with the obligations of one or more of nuclear non-proliferation

agreement.

• Support of international efforts towards non-proliferation of weapons of mass

destruction and of their delivery vehicle.

• Presently, India is not a member the group.

Why the membership is important for India?

• Membership will increase India’s access to state-of-the-art technology from the

other members of the Group.

Page 9: SIMPLYFYING IAS EXAM PREPARATION - INSIGHTSIAS · 2020. 4. 7. · Insta 75 Days Revision Plan for UPSC Civil Services Prelims – 2020 This document is the compilation of 100 questions

INSTA 75 Days REVISION PLAN for Prelims 2020 - InstaTests

www.insightsonindia.com 7 Insights IAS

• Access to technology and being allowed to produce nuclear equipment will give

a boost to the Make in India program. That will, in turn, boost the economic

growth of our country.

• As per India’s INDC under the Paris Climate agreement, we have committed to

reducing dependence on fossil fuels and ensuring that 40% of its energy is

sourced from renewable and clean sources. In order to achieve this target, we

need to scale up nuclear power production. This can only happen if India gains

access to the NSG.

• India will get an opportunity to voice it’s concern if in case of change in the

provision of the NSG guidelines.

Other Benefits associated with NSG membership- Once admitted, an NSG member

state gets:

• Timely information on nuclear matters.

• Contributes by way of information.

• Has confirmed credentials.

• Can act as an instrument of harmonization and coordination.

• Is part of a very transparent process.

https://www.insightsonindia.com/2019/09/26/nuclear-suppliers-group-2/

7. Consider the following statements regarding Primary activities

1. Primary activities are directly dependent on environment as these refer

to utilization of earth’s resources

2. It includes hunting and gathering, fishing, agriculture, and mining and

quarrying.

Which of the statements given above is/are correct?

(a) 1 only

(b) 2 only

(c) Both 1 and 2

(d) Neither 1 nor 2

Solution: C

Human activities which generate income are known as economic activities.

Economic activities are broadly grouped into primary, secondary, tertiary and

quaternary activities.

1. Primary activities are directly dependent on environment as these refer to

utilisation of earth’s resources such as land, water, vegetation, building

materials and minerals. It, thus includes, hunting and gathering, pastoral

activities, fishing, forestry, agriculture, and mining and quarrying.

Page 10: SIMPLYFYING IAS EXAM PREPARATION - INSIGHTSIAS · 2020. 4. 7. · Insta 75 Days Revision Plan for UPSC Civil Services Prelims – 2020 This document is the compilation of 100 questions

INSTA 75 Days REVISION PLAN for Prelims 2020 - InstaTests

www.insightsonindia.com 8 Insights IAS

2. Secondary activities add value to natural resources by transforming raw

materials into valuable products. Cotton in the boll has limited use but after it is

transformed into yarn, becomes more valuable and can be used for making

clothes. Iron ore, cannot be used; directly from the mines, but after being

converted into steel it gets its value and can be used for making many valuable

machines, tools, etc.

o Secondary activities, therefore, are concerned with manufacturing,

processing and construction (infrastructure) industries.

3. Tertiary activities are related to the service sector. Manpower is an important

component of the service sector as most of the tertiary activities are performed

by skilled labour, professionally trained experts and consultants.

o Tertiary activities include both production and exchange. The production

involves the ‘provision’ of services that are ‘consumed’. The output is

indirectly measured in terms of wages and salaries. Exchange, involves

trade, transport and communication facilities that are used to overcome

distance. Tertiary activities, therefore, involve the commercial output of

services rather than the production of tangible goods. They are not

directly involved in the processing of physical raw materials. Common

examples are the work of a plumber, electrician, technician, launderer,

barber, shopkeeper, driver, cashier, teacher, doctor, lawyer and

publisher etc. The main difference between secondary activities and

tertiary activities is that the expertise provided by services relies more

heavily on specialised skills, experience and knowledge of the workers

rather than on the production techniques, machinery and factory

processes.

8. Consider the following statements regarding forms of the settlements

1. Rectangular pattern: Such patterns of rural settlements are found where

the roads are rectangular and cut each other at right angles.

2. Circular pattern: Such patterns of rural settlements are found where

several roads converge

3. Star like pattern: Villages develop around lakes, tanks

Which of the statements given above is/are correct?

(a) 1 only

(b) 1 and 3 only

(c) 2 only

(d) 1, 2 and 3

Solution: A

Page 11: SIMPLYFYING IAS EXAM PREPARATION - INSIGHTSIAS · 2020. 4. 7. · Insta 75 Days Revision Plan for UPSC Civil Services Prelims – 2020 This document is the compilation of 100 questions

INSTA 75 Days REVISION PLAN for Prelims 2020 - InstaTests

www.insightsonindia.com 9 Insights IAS

Forms or shapes of the settlements: These may be a number of geometrical forms

and shapes such as Linear, rectangular, circular star like, T-shaped village, double

village, cross-shaped village etc.

(a) Linear pattern: In such settlements houses are located along a road, railway line,

river, canal edge of a valley or along a levee.

(b) Rectangular pattern: Such patterns of rural settlements are found in plain areas or

wide inter montane valleys. The roads are rectangular and cut each other at right

angles.

(c) Circular pattern: Circular villages develop around lakes, tanks and sometimes the

village is planned in such a way that the central part remains open and is used for

keeping the animals to protect them from wild animals.

(d) Star like pattern: Where several roads converge, star shaped settlements develop

by the houses built along the roads.

(e) T-shaped, Y-shaped, Cross-shaped or cruciform settlements: T -shape

settlements develop at tri-junctions of the roads, while Y-shaped settlements

emerge as the places where two roads converge on the third one and houses are

built along these roads. Cruciform settlements develop on the cross-roads and

houses extend in all the four direction.

(f) Double village: These settlements extend on both sides of a river where there is a

bridge or a ferry.

9. Consider the following statements regarding National Animal Disease Control

Programme.

1. It is a 100% centrally funded programme

2. It aims to eradicate Foot and Mouth Disease and Brucellosis by 2025.

Which of the statements given above is/are correct?

(a) 1 only

(b) 2 only

(c) Both 1 and 2

(d) Neither 1 nor 2

Solution: A

PM to launch National Animal Disease Control Programme for Foot and Mouth

Disease.

• It is a 100% centrally funded programme, with a total outlay of Rs.12,652 crore

from 2019 to 2024.

• It aims to control Foot and Mouth Disease and Brucellosis by 2025 with

vaccination and eventual eradication by 2030.

Page 12: SIMPLYFYING IAS EXAM PREPARATION - INSIGHTSIAS · 2020. 4. 7. · Insta 75 Days Revision Plan for UPSC Civil Services Prelims – 2020 This document is the compilation of 100 questions

INSTA 75 Days REVISION PLAN for Prelims 2020 - InstaTests

www.insightsonindia.com 10 Insights IAS

About Foot-and-mouth disease:

• It is an infectious and sometimes fatal viral disease.

• Affects cloven-hoofed animals, including domestic and wild bovids.

• Symptoms: The virus causes a high fever for two or three days, followed by

blisters inside the mouth and on the feet that may rupture and cause lameness.

• The virus responsible for the disease is a picornavirus, the prototypic member

of the genus Aphthovirus.

Spread of the disease and Concerns associated:

• Foot-and-mouth disease (FMD) has severe implications for animal farming,

since it is highly infectious and can be spread by infected animals through

aerosols, through contact with contaminated farming equipment, vehicles,

clothing, or feed, and by domestic and wild predators.

Can Humans be affected?

• Humans can be infected with foot-and-mouth disease through contact with

infected animals, but this is extremely rare. Some cases were caused by

laboratory accidents. Because the virus that causes FMD is sensitive to

stomach acid, it cannot spread to humans via consumption of infected meat,

except in the mouth before the meat is swallowed. Symptoms of FMD in

humans include malaise, fever, vomiting, red ulcerative lesions (surface-

eroding damaged spots) of the oral tissues, and sometimes vesicular lesions

(small blisters) of the skin.

https://www.insightsonindia.com/2019/09/07/foot-and-mouth-disease/

10. Consider the following pairs regarding different names of Shifting cultivation

and their region

1. Jhuming - North eastern states of India 2. Milpa - Indonesia and Malaysia 3. Ladang - Central America and Mexico

Which of the pairs given above is/are correctly matched?

(a) 1 only

(b) 2 and 3 only

(c) 1 and 3 only

(d) 1, 2 and 3

Solution: A

• The vegetation is usually cleared by fire, and the ashes add to the fertility of the

soil. Shifting cultivation is thus, also called slash and burn agriculture. The

cultivated patches are very small and cultivation is done with very primitive tools

Page 13: SIMPLYFYING IAS EXAM PREPARATION - INSIGHTSIAS · 2020. 4. 7. · Insta 75 Days Revision Plan for UPSC Civil Services Prelims – 2020 This document is the compilation of 100 questions

INSTA 75 Days REVISION PLAN for Prelims 2020 - InstaTests

www.insightsonindia.com 11 Insights IAS

such as sticks and hoes. After sometime (3 to 5 years) the soil loses its fertility

and the farmer shifts to another parts and clears other patch of the forest for

cultivation. The farmer may return to the earlier patch after sometime. One of

the major problems of shifting cultivation is that the cycle of jhum becomes less

and less due to loss of fertility in different parcels.

• It is prevalent in tropical region in different names, e.g. Jhuming in North

eastern states of India, Milpa in central America and Mexico and Ladang

in Indonesia and Malaysia.

11. Consider the following statements regarding different types of port

1. Ports of Call - These ports serve warships and have repair workshops for them

2. Packet Station - These are exclusively concerned with the transportation of passengers and mail across water bodies covering short distances

3. Entrepot Ports - These are collection centres where the goods are brought from different countries for export

Which of the statements given above is/are correct?

(a) 1 only

(b) 2 and 3 only

(c) 1 and 3 only

(d) 1, 2 and 3

Solution: B

Types of port on the basis of specialized functions:

(i) Oil Ports: These ports deal in the processing and shipping of oil. Some of

these are tanker ports and some are refinery ports. Maracaibo in Venezuela,

Esskhira in Tunisia, Tripoli in Lebanon

(ii) Ports of Call: These are the ports which originally developed as calling points

on main sea routes where ships used to anchor for refuelling, watering and

taking food items. Later on, they developed into commercial ports. Aden,

Honolulu and Singapore are good examples.

(iii) Packet Station: These are also known as ferry ports. These packet stations

are exclusively concerned with the transportation of passengers and mail

across water bodies covering short distances. These stations occur in pairs

Page 14: SIMPLYFYING IAS EXAM PREPARATION - INSIGHTSIAS · 2020. 4. 7. · Insta 75 Days Revision Plan for UPSC Civil Services Prelims – 2020 This document is the compilation of 100 questions

INSTA 75 Days REVISION PLAN for Prelims 2020 - InstaTests

www.insightsonindia.com 12 Insights IAS

located in such a way that they face each other across the water body, e.g.

Dover in England and Calais in France across the English Channel.

(iv) Entrepot Ports: These are collection centres where the goods are brought from

different countries for export. Singapore is an entrepot for Asia. Rotterdam for

Europe, and Copenhagen for the Baltic region.

(v) Naval Ports: These are ports which have only strategic importance. These

ports serve warships and have repair workshops for them. Kochi and Karwar

are examples of such ports in India.

12. The Motihari-Amalekhgunj petroleum pipeline is a joint project between:

(a) India and China

(b) India and Bangladesh

(c) India and Nepal

(d) China and Nepal

Solution: C

• PM Modi and his Nepalese counterpart KPS Oli will “switch on” the Motihari-

Amalekhgunj petroleum pipeline from their offices in New Delhi and

Kathmandu.

• The pipeline will transport fuel from Barauni refinery in Bihar’s Begusarai district

to Amalekhgunj in southeastern Nepal, situated across the border from Raxaul

in East Champaran district. The 69-km pipeline will drastically reduce the cost

of transporting fuel to landlocked Nepal from India.

• The pipeline will help in tackling the oil storage problem in Nepal and doing

away with transportation of petroleum products through tankers.

13. Consider the following statements regarding Ageing Population

1. Population ageing is the process by which the share of the older

population becomes proportionally larger.

2. It is very common in least developed countries.

Which of the statements given above is/are correct?

(a) 1 only

(b) 2 only

(c) Both 1 and 2

(d) Neither 1 nor 2

Solution: A

Page 15: SIMPLYFYING IAS EXAM PREPARATION - INSIGHTSIAS · 2020. 4. 7. · Insta 75 Days Revision Plan for UPSC Civil Services Prelims – 2020 This document is the compilation of 100 questions

INSTA 75 Days REVISION PLAN for Prelims 2020 - InstaTests

www.insightsonindia.com 13 Insights IAS

Ageing Population

• Population ageing is the process by which the share of the older population

becomes proportionally larger. This is a new phenomenon of the twentieth

century. In most of the developed countries of the world, population in higher

age groups has increased due to increased life expectancy. With a

reduction in birth rates, the proportion of children in the population has declined.

14. Which of the following countries does not share its border with Italy

(a) Spain

(b) France

(c) Switzerland

(d) Austria

Solution: A

Page 16: SIMPLYFYING IAS EXAM PREPARATION - INSIGHTSIAS · 2020. 4. 7. · Insta 75 Days Revision Plan for UPSC Civil Services Prelims – 2020 This document is the compilation of 100 questions

INSTA 75 Days REVISION PLAN for Prelims 2020 - InstaTests

www.insightsonindia.com 14 Insights IAS

15. Consider the following statements regarding Deucha Panchami block

1. It is India’s largest iron ore block.

2. It is situated in Odisha.

Which of the statements given above is/are correct?

(a) 1 only

(b) 2 only

(c) Both 1 and 2

(d) Neither 1 nor 2

Solution: D

World’s Second Largest Coal Block:

• Deaucha Panchami coal block of Birbhum Coalfield Area is World’s Second

Largest Coal Block.

• Situated in West Bengal. This coal mine is the largest coal mine or coal block

in Asia, due to the number of coal reserves.

https://www.insightsonindia.com/2019/09/21/insights-daily-current-affairs-pib-21-

september-2019/

Page 17: SIMPLYFYING IAS EXAM PREPARATION - INSIGHTSIAS · 2020. 4. 7. · Insta 75 Days Revision Plan for UPSC Civil Services Prelims – 2020 This document is the compilation of 100 questions

INSTA 75 Days REVISION PLAN for Prelims 2020 - InstaTests

www.insightsonindia.com 15 Insights IAS

16. Foot Loose Industries are those which

(a) Provide Flexible working hours for labourers

(b) Depend on components which are available anywhere

(c) Dependent of weight losing materials

(d) are mobile in nature.

Solution: B

Foot Loose Industries

• Foot loose industries can be located in a wide variety of places. They are not

dependent on any specific raw material, weight losing or otherwise.

• They largely depend on component parts which can be obtained anywhere.

They produce in small quantity and also employ a small labour force. These are

generally not polluting industries. The important factor in their location is

accessibility by road network.

Extra Learning:

Page 18: SIMPLYFYING IAS EXAM PREPARATION - INSIGHTSIAS · 2020. 4. 7. · Insta 75 Days Revision Plan for UPSC Civil Services Prelims – 2020 This document is the compilation of 100 questions

INSTA 75 Days REVISION PLAN for Prelims 2020 - InstaTests

www.insightsonindia.com 16 Insights IAS

17. Which of the following countries does not share its border with South Atlantic

Ocean

(a) Ghana

(b) Republic of the Congo

(c) Nigeria

(d) Niger

Solution: D

18. Consider the following statements regarding recent changes made with respect

to Cabinet secretary

1. He is appointed for a fixed tenure of five years.

2. He is under the direct charge of the Ministry of Parliamentary Affairs

Which of the statements given above is/are correct?

(a) 1 only

(b) 2 only

(c) Both 1 and 2

(d) Neither 1 nor 2

Page 19: SIMPLYFYING IAS EXAM PREPARATION - INSIGHTSIAS · 2020. 4. 7. · Insta 75 Days Revision Plan for UPSC Civil Services Prelims – 2020 This document is the compilation of 100 questions

INSTA 75 Days REVISION PLAN for Prelims 2020 - InstaTests

www.insightsonindia.com 17 Insights IAS

Solution: D

• Govt Amends 60-Year-Old rule to pave way for Cabinet Secretary extension.

With this, the current Cabinet Secretary Pradeep Kumar Sinha, has become

the longest-serving bureaucrat in the post in the country’s history.

Key concept- changes made:

• A cabinet secretary is appointed for a fixed tenure of two years.

• According to All India Services (Death-Cum-Retirement-Benefits) Rules,

1958, the government can give extension in service to a cabinet secretary

provided the total tenure does not exceed four years.

• As per the modified rules, the central government may give an extension in

service for a further period not exceeding three months, beyond the period of

four years to a cabinet secretary.

Role of the cabinet secretary:

• The cabinet secretariat is under the direct charge of the prime minister.

• The administrative head of the secretariat is the cabinet secretary who is also

the ex-officio chairman of the civil services board.

Functions:

• The cabinet secretariat assists in decision-making in government by ensuring

inter-ministerial coordination, ironing out differences amongst ministries or

departments and evolving consensus through the instrumentality of the

standing or ad hoc committees of secretaries.

• Management of major crisis situations in the country and coordinating activities

of various ministries in such a situation is also one of the functions of the cabinet

secretariat.

• Cabinet Secretariat is responsible for the administration of the Government of

India (Transaction of Business) Rules, 1961 and the Government of India

(Allocation of Business) Rules 1961, facilitating smooth transaction of

business in Ministries/ Departments of the Government.

https://www.insightsonindia.com/2019/06/10/cabinet-secretary/

19. Consider the following statements

1. Quaternary activities center around research, development and may be

seen as an advanced form of services involving specialized knowledge

and technical skills.

2. The highest level of decision makers or policy makers performs quinary

activities.

Which of the statements given above is/are correct?

(a) 1 only

(b) 2 only

Page 20: SIMPLYFYING IAS EXAM PREPARATION - INSIGHTSIAS · 2020. 4. 7. · Insta 75 Days Revision Plan for UPSC Civil Services Prelims – 2020 This document is the compilation of 100 questions

INSTA 75 Days REVISION PLAN for Prelims 2020 - InstaTests

www.insightsonindia.com 18 Insights IAS

(c) Both 1 and 2

(d) Neither 1 nor 2

Solution: C

• Quaternary activities involve some of the following: the collection, production

and dissemination of information or even the production of information.

Quaternary activities centre around research, development and may be seen

as an advanced form of services involving specialized knowledge and technical

skills.

QUINARY ACTIVITIES

• The highest level of decision makers or policy makers perform quinary

activities. These are subtly different from the knowledge-based industries that

the quinary sector in general deals with.

• Quinary activities are services that focus on the creation, re-arrangement and

interpretation of new and existing ideas; data interpretation and the use and

evaluation of new technologies. Often referred to as ‘gold collar’ professions,

they represent another subdivision of the tertiary sector representing special

and highly paid skills of senior business executives, government officials,

research scientists, financial and legal consultants, etc. Their importance in the

structure of advanced economies far outweighs their numbers.

20. Consider the following statements regarding Nipah Virus

1. The natural host of the virus is fruit bats belonging to the family

Pteropodidae.

2. The incubation period of the virus is 7-14 days.

3. The virus can be transmitted to humans from animals.

Which of the statements given above is/are correct?

(a) 2 and 3 only

(b) 1 and 3 only

(c) 1 and 2 only

(d) 1, 2 and 3

Solution: D

• The deadly Nipah Virus has resurfaced in Kerala.

Page 21: SIMPLYFYING IAS EXAM PREPARATION - INSIGHTSIAS · 2020. 4. 7. · Insta 75 Days Revision Plan for UPSC Civil Services Prelims – 2020 This document is the compilation of 100 questions

INSTA 75 Days REVISION PLAN for Prelims 2020 - InstaTests

www.insightsonindia.com 19 Insights IAS

What is Nipah Virus?

• According to WHO, the Nipah virus infection is a newly emerging zoonosis, that

is, a disease transmitted from animals to humans. The virus belongs to a new

genus termed Henipavirus (subfamily Paramyxovirinae).

• The natural host of the virus are fruit bats belonging to the family Pteropodidae.

In 2004, humans were affected after eating the date palm contaminated by

infected fruit bats. Pigs can also act as intermediate hosts.

• The virus can be transmitted to humans from animals (such as bats or pigs).

• The incubation period of the virus is 7-14 days

When it was first reported?

• It was first identified in 1998 at Kampung Sungai Nipah village, Malaysia. The

virus is named after this village.

What are the symptoms in humans?

• The symptoms of Nipah are similar to that of influenza: fever, muscle pain, and

respiratory problems. Inflammation of the brain can also cause disorientation.

Late onset of Encephalitis can also occur. Sometimes a person can have an

asymptomatic infection, and be a carrier of Nipah and not show any symptoms.

Are there any vaccines?

• Currently, there are no vaccines for both humans and animals. Intensive

supportive care is given to humans infected by Nipah virus. According to WHO,

ribavarin can reduce the symptoms of nausea, vomiting, and convulsions

associated with the disease. Individuals infected need to be hospitalised and

isolated. Special care should be taken to prevent human-to-human

transmission. Surveillance systems should be established to detect the virus

quickly and to initiate appropriate control measures.

https://www.insightsonindia.com/2019/06/04/nipah-virus/

21. Consider the following pairs regarding Age-Sex Pyramid with their

characteristics:

1. Expanding Populations

: triangular shaped pyramid

2. Constant Population

: bell shaped and tapered towards the top

3. Declining Populations

: narrow base and a tapered top

Which of the pairs given above is/are correctly matched?

(a) 1 and 2 only

(b) 2 and 3 only

(c) 1 and 3 only

Page 22: SIMPLYFYING IAS EXAM PREPARATION - INSIGHTSIAS · 2020. 4. 7. · Insta 75 Days Revision Plan for UPSC Civil Services Prelims – 2020 This document is the compilation of 100 questions

INSTA 75 Days REVISION PLAN for Prelims 2020 - InstaTests

www.insightsonindia.com 20 Insights IAS

(d) 1, 2 and 3

Solution: D

Age-Sex Pyramid

• The age-sex structure of a population refers to the number of females and

males in different age groups. A population pyramid is used to show the age-

sex structure of the population.

• The shape of the population pyramid reflects the characteristics of the

population. The left side shows the percentage of males while the right side

shows the percentage of women in each age group.

Expanding Populations

• The age-sex pyramid of Nigeria as you can see is a triangular shaped pyramid

with a wide base and is typical of less developed countries.

• These have larger populations in lower age groups due to high birth rates. If

you construct the pyramids for Bangladesh and Mexico, it would look the same.

Constant Population

• Australia’s age-sex pyramid is bell shaped and tapered towards the top. This

shows birth and death rates are almost equal leading to a near constant

population.

Page 23: SIMPLYFYING IAS EXAM PREPARATION - INSIGHTSIAS · 2020. 4. 7. · Insta 75 Days Revision Plan for UPSC Civil Services Prelims – 2020 This document is the compilation of 100 questions

INSTA 75 Days REVISION PLAN for Prelims 2020 - InstaTests

www.insightsonindia.com 21 Insights IAS

Declining Populations

• The Japan pyramid has a narrow base and a tapered top showing low birth and

death rates. The population growth in developed countries is usually zero or

negative

22. Project REPLAN (REducing PLAstic in Nature) is program under

(a) Khadi and Village Industries Commission (KVIC)

(b) Central Pollution Control Board (CPCB)

(c) Council of Scientific and Industrial Research (CSIR)

(d) Ministry of Environment

Solution: A

Page 24: SIMPLYFYING IAS EXAM PREPARATION - INSIGHTSIAS · 2020. 4. 7. · Insta 75 Days Revision Plan for UPSC Civil Services Prelims – 2020 This document is the compilation of 100 questions

INSTA 75 Days REVISION PLAN for Prelims 2020 - InstaTests

www.insightsonindia.com 22 Insights IAS

KVIC, as part of its commitment to Swachh Bharat Abhiyaan, had started

manufacturing of plastic-mixed handmade paper under its project REPLAN

(REducing PLAstic in Nature).

• In this project, the waste plastic is collected, cleaned, chopped, beaten and

treated for softness.

• After that, it is mixed with the paper raw material i.e. cotton rags pulp in a ratio

of 80 % (pulp) and 20% (plastic waste).

23. Consider the following pairs regarding railway lines with the places they connect

1. Trans-Siberian railways

: St. Petersburg to Vladivostok

2. Trans-Canadian railways

: Victoria to Halifax

3. The Australian Trans–Continental Railway

: Perth to Sydney

4. The Orient Express : Paris to Istanbul Which of the pairs given above is/are correctly matched?

(a) 1, 2 and 3 only

(b) 2, 3 and 4 only

(c) 1, 3 and 4 only

(d) 1, 2, 3 and 4

Solution: D

Page 25: SIMPLYFYING IAS EXAM PREPARATION - INSIGHTSIAS · 2020. 4. 7. · Insta 75 Days Revision Plan for UPSC Civil Services Prelims – 2020 This document is the compilation of 100 questions

INSTA 75 Days REVISION PLAN for Prelims 2020 - InstaTests

www.insightsonindia.com 23 Insights IAS

Page 26: SIMPLYFYING IAS EXAM PREPARATION - INSIGHTSIAS · 2020. 4. 7. · Insta 75 Days Revision Plan for UPSC Civil Services Prelims – 2020 This document is the compilation of 100 questions

INSTA 75 Days REVISION PLAN for Prelims 2020 - InstaTests

www.insightsonindia.com 24 Insights IAS

The Australian Trans–Continental Railway:

• This rail-line runs west-east across the southern part of the continent from

Perth on the west coast, to Sydney on the east coast. Passing through

Kalgoorlie, Broken Hill and Port Augusta

The Orient Express:

• This line runs from Paris to Istanbul passing through Strasbourg, Munich,

Vienna, Budapest and Belgrade.

24. Integrated Tiger Habitat Conservation Programme (ITHPC) is an important

program under

(a) The International Union for Conservation of Nature (IUCN)

(b) The Convention on International Trade in Endangered Species of Wild

Fauna and Flora (CITES)

(c) Global Tiger Forum (GTF)

(d) The World Wide Fund for Nature (WWF)

Solution: A

Union Environment Ministry released a report on Status of Tiger Habitats in high

altitude ecosystems.

• The study is led by the Global Tiger Forum (GTF), with range country

governments of Bhutan, India and Nepal, along with WWF. It has been

supported by the Integrated Tiger Habitat Conservation Programme

(ITHPC) of the IUCN.

• Initiated in 2014, the Integrated Tiger Habitat Conservation Programme

(ITHCP) or ‘Tiger Programme‘ is a grant-making initiative which contributes to

the Global Tiger Recovery Programme (GTRP), a global effort to double tiger

numbers in the wild by 2022. The programme consists of a portfolio of 12 large-

scale projects in key Tiger Conservation Landscapes across Bangladesh,

Bhutan, India, Indonesia, Nepal and Myanmar.

The Tiger Programme is based on three pillars:

1. Protecting tiger species and their prey from the threat of poaching;

2. Preserving tiger habitats, including core habitats, buffer zones and corridors;

3. Supporting human populations living in tiger landscapes.

https://www.iucn.org/theme/species/our-work/action-ground/integrated-tiger-habitat-

conservation-programme

Page 27: SIMPLYFYING IAS EXAM PREPARATION - INSIGHTSIAS · 2020. 4. 7. · Insta 75 Days Revision Plan for UPSC Civil Services Prelims – 2020 This document is the compilation of 100 questions

INSTA 75 Days REVISION PLAN for Prelims 2020 - InstaTests

www.insightsonindia.com 25 Insights IAS

25. Consider the following statements regarding the Plastic Waste Management

Rules, 2016:

1. It aims to increase minimum thickness of plastic carry bags from 40 to

80 microns.

2. Rural areas have been brought under the ambit of these Rules.

3. Persons organising public events (marriage functions, religious

gatherings, public meetings etc) have been made responsible for

management of waste generated from these events.

Which of the statements given above is/are correct?

(a) 1 and 3 only

(b) 2 and 3 only

(c) 2 only

(d) 1 only

Solution: B

• The Indian Army has removed 130 tonnes of garbage from the Siachen Glacier

and is cutting potential trash in rations.

• To address the issue of scientific plastic waste management, the Plastic Waste

(Management and Handling) Rules, 2011 were notified in 2011, which included

plastic waste management. The Government has notified the Plastic Waste

Management Rules, 2016, in suppression of the earlier Plastic Waste

(Management and Handling) Rules, 2011.

The Plastic Waste Management Rules, 2016 aim to:

• Increase minimum thickness of plastic carry bags from 40 to 50 microns and

stipulate minimum thickness of 50 micron for plastic sheets also to facilitate

collection and recycle of plastic waste.

• Expand the jurisdiction of applicability from the municipal area to rural areas,

because plastic has reached rural areas also

• To bring in the responsibilities of producers and generators, both in plastic

waste management system and to introduce collect back system of plastic

waste by the producers/brand owners, as per extended producers responsibility

• To introduce collection of plastic waste management fee through pre-

registration of the producers, importers of plastic carry bags/multilayered

packaging and vendors selling the same for establishing the waste

management system

• To promote use of plastic waste for road construction as per Indian Road

Congress guidelines or energy recovery, or waste to oil etc. for gainful utilization

of waste and also address the waste disposal issue; to entrust more

responsibility on waste generators, namely payment of user charge as

prescribed by local authority, collection and handing over of waste by the

institutional generator, event organizers.

Page 28: SIMPLYFYING IAS EXAM PREPARATION - INSIGHTSIAS · 2020. 4. 7. · Insta 75 Days Revision Plan for UPSC Civil Services Prelims – 2020 This document is the compilation of 100 questions

INSTA 75 Days REVISION PLAN for Prelims 2020 - InstaTests

www.insightsonindia.com 26 Insights IAS

• An eco-friendly product, which is a complete substitute of the plastic in all uses,

has not been found till date. In the absence of a suitable alternative, it is

impractical and undesirable to impose a blanket ban on the use of plastic all

over the country. The real challenge is to improve plastic waste management

systems.

What’s new in Plastic Waste Management Rules, 2016

• Rural areas have been brought in ambit of these Rules since plastic has

reached to rural areas also. Responsibility for implementation of the rules is

given to Gram Panchayat.

• First time, responsibility of waste generators is being introduced. Individual

and bulk generators like offices, commercial establishments, industries are to

segregate the plastic waste at source, handover segregated waste, pay user

fee as per bye-laws of the local bodies.

• Plastic products are left littered after the public events (marriage functions,

religious gatherings, public meetings etc) held in open spaces. First time,

persons organising such events have been made responsible for management

of waste generated from these events.

• Use of plastic sheet for packaging, wrapping the commodity except those

plastic sheet’s thickness, which will impair the functionality of the product are

brought under the ambit of these rules. A large number of commodities are

being packed/wrapped in to plastic sheets and thereafter such sheets are left

for littered. Provisions have been introduced to ensure their collection and

channelization to authorised recycling facilities.

• Extended Producer Responsibility: Earlier, EPR was left to the discretion of

the local bodies. First time, the producers (i.e persons engaged in manufacture,

or import of carry bags, multi-layered packaging and sheets or like and the

persons using these for packaging or wrapping their products) and brand

owners have been made responsible for collecting waste generated from their

products. They have to approach local bodies for formulation of plan/system for

the plastic waste management within the prescribed timeframe.

• State Pollution Control Board (SPCBs) will not grant/renew registration of

plastic bags, or multi-layered packaging unless the producer proposes the

action plan endorsed by the concerned State Development Department.

• Producers to keep a record of their vendors to whom they have supplied raw

materials for manufacturing carry bags, plastic sheets, and multi-layered

packaging. This is to curb manufacturing of these products in unorganised

sector.

• The entry points of plastic bags/plastic sheets/multi-layered packaging in to

commodity supply chain are primarily the retailers and street vendors. They

have been assigned the responsibility of not to provide the commodities in

plastic bags/plastic sheets/multi-layered packaging which do not conform to

these rules. Otherwise, they will have to pay the fine.

• Plastic carry bag will be available only with shopkeepers/street vendors pre-

registered with local bodies on payment of certain registration fee. The amount

Page 29: SIMPLYFYING IAS EXAM PREPARATION - INSIGHTSIAS · 2020. 4. 7. · Insta 75 Days Revision Plan for UPSC Civil Services Prelims – 2020 This document is the compilation of 100 questions

INSTA 75 Days REVISION PLAN for Prelims 2020 - InstaTests

www.insightsonindia.com 27 Insights IAS

collected as registration fee by local bodies is to be used for waste

management.

• Central Pollution Control Board (CPCB) has been mandated to formulate the

guidelines for thermoset plastic (plastic difficult to recycle). In the earlier Rules,

there was no specific provision for such type of plastic.

• Manufacturing and use of non-recyclable multi-layered plastic to be phased in

two years.

Plastic Waste Management (Amendment) Rules 2018:

• The Ministry of Environment, Forest and Climate Change has notified the

Plastic Waste Management (Amendment) Rules 2018 on March 27, 2018.

• The amended Rules lay down that the phasing out of Multilayered Plastic

(MLP) is now applicable to MLP, which are “non-recyclable, or non-energy

recoverable, or with no alternate use.”

• The amended Rules also prescribe a central registration system for the

registration of the producer/importer/brand owner. The Rules also lay down that

any mechanism for the registration should be automated and should take into

account ease of doing business for producers, recyclers and manufacturers.

The centralised registration system will be evolved by Central Pollution Control

Board (CPCB) for the registration of the producer/importer/brand owner. While

a national registry has been prescribed for producers with presence in more

than two states, a state-level registration has been prescribed for smaller

producers/brand owners operating within one or two states.

DAY – 14 (InstaTest-14)

26. Consider the following statements regarding the alluvial plains

1. Bhangar is a narrow belt parallel to the Shiwalik foothills where the

streams and rivers coming from the mountains disappear.

2. In Tarai belt, most of the streams and rivers re-emerge and create

marshy and swampy conditions.

Which of the statements given above is/are correct?

(a) 1 only

(b) 2 only

(c) Both 1 and 2

(d) Neither 1 nor 2

Solution: B

Page 30: SIMPLYFYING IAS EXAM PREPARATION - INSIGHTSIAS · 2020. 4. 7. · Insta 75 Days Revision Plan for UPSC Civil Services Prelims – 2020 This document is the compilation of 100 questions

INSTA 75 Days REVISION PLAN for Prelims 2020 - InstaTests

www.insightsonindia.com 28 Insights IAS

From the north to the south, Northern Plains can be divided into three major zones:

the Bhabar, the Tarai and the alluvial plains. The alluvial plains can be further divided

into the Khadar and the Bhangar.

Bhabar is a narrow belt ranging between 8-10 km parallel to the Shiwalik foothills at

the break-up of the slope. As a result of this, the streams and rivers coming from the

mountains deposit heavy materials of rocks and boulders, and at times, disappear in

this zone.

South of the Bhabar is the Tarai belt, with an approximate width of 10-20 km where

most of the streams and rivers re-emerge without having any properly demarcated

channel, thereby, creating marshy and swampy conditions known as the Tarai.

This has a luxurious growth of natural vegetation and houses a varied wildlife. The

south of Tarai is a belt consisting of old and new alluvial deposits known as the

Bhangar and Khadar respectively.

27. Consider the following statements regarding River Jhelum

1. Srinagar is located on the banks of Jhelum river.

2. Jhelum is in its mature stage and forms meanders in the Kashmir valley.

Which of the statements given above is/are correct?

(a) 1 only

(b) 2 only

(c) Both 1 and 2

(d) Neither 1 nor 2

Solution: A

Srinagar, capital city of the state of Jammu and Kashmir is located on the banks of

Jhelum river. Dal Lake in Srinagar presents an interesting physical feature.

Jhelum in the valley of Kashmir is still in its youth stage and yet forms meanders – a

typical feature associated with the mature stage in the evolution of fluvial land form.

28. Consider the following statements regarding Oxytocin

1. It acts both as a hormone and as a brain neurotransmitter.

2. The release of oxytocin regulate female reproductive functions namely

Childbirth and Breast-feeding.

Which of the statements given above is/are correct?

(a) 1 only

(b) 2 only

Page 31: SIMPLYFYING IAS EXAM PREPARATION - INSIGHTSIAS · 2020. 4. 7. · Insta 75 Days Revision Plan for UPSC Civil Services Prelims – 2020 This document is the compilation of 100 questions

INSTA 75 Days REVISION PLAN for Prelims 2020 - InstaTests

www.insightsonindia.com 29 Insights IAS

(c) Both 1 and 2

(d) Neither 1 nor 2

Solution: C

About Oxytocin:

• Oxytocin has also been dubbed the hug hormone, cuddle chemical, moral

molecule, and the bliss hormone due to its effects on behavior, including its role

in love and in female reproductive biological functions in reproduction.

• Oxytocin is a hormone that is made in the brain, in the hypothalamus. It is

transported to, and secreted by, the pituitary gland, which is located at the

base of the brain.

• It acts both as a hormone and as a brain neurotransmitter.

• The release of oxytocin by the pituitary gland acts to regulate two female

reproductive functions: Childbirth and Breast-feeding.

Extra Reading:

Indian Medical Association has said Karnataka Antibiotics and Pharmaceuticals Ltd

(KAPL) neither has the experience nor the capacity to handle Oxytocin production.

What’s the issue?

• The Delhi high court had quashed the Centre’s December 14, 2018 notification,

which had banned its sale by private manufacturers and retail chemists, saying

the sale was allowed. Essentially, this meant that only KAPL could produce the

drug as there is no other public sector enterprise doing so. However, Delhi high

court quashed the amended order too. The central government moved

Supreme Court against the Delhi high court order.

What’s the concern now?

• KAPL has said bulk production of the drug would take three-four years. This

would put lives of many pregnant women at risk as it would lead to acute

shortages.

• With the ban, the government did not adequately weigh in the danger of its

order to the users of oxytocin, nor consider the deleterious effect of possible

restricted supply if manufacture is confined to one unit on pregnant women and

young mothers.

29. Consider the following statements regarding Dooars or Duars

1. Duars are the longitudinal valley lying between lesser Himalayas and

shiwaliks

2. The Duars are found in northeastern India.

Page 32: SIMPLYFYING IAS EXAM PREPARATION - INSIGHTSIAS · 2020. 4. 7. · Insta 75 Days Revision Plan for UPSC Civil Services Prelims – 2020 This document is the compilation of 100 questions

INSTA 75 Days REVISION PLAN for Prelims 2020 - InstaTests

www.insightsonindia.com 30 Insights IAS

Which of the statements given above is/are correct?

(a) 1 only

(b) 2 only

(c) Both 1 and 2

(d) Neither 1 nor 2

Solution: B

Duar formations

• As compared to the other sections of the Himalayas, these along with the

Arunachal Himalayas are conspicuous by the absence of the Shiwalik

formations.

• In place of the Shiwaliks here, the ‘duar formations’ are important, which have

also been used for the development of tea gardens.

• The Dooars or Duars are the alluvial floodplains in northeastern India that

lie south of the outer foothills of the Himalayas and north of the Brahmaputra

River basin.

• The Duars are found in northeastern India.

30. Arrange the following hills from south to north

1. Dafla Hills

2. Mizo Hills

3. Naga Hills

Which of the following order given below is correct?

(a) 1 3 2

(b) 1 2 3

(c) 3 1 2

(d) 2 3 1

Solution: D

Page 33: SIMPLYFYING IAS EXAM PREPARATION - INSIGHTSIAS · 2020. 4. 7. · Insta 75 Days Revision Plan for UPSC Civil Services Prelims – 2020 This document is the compilation of 100 questions

INSTA 75 Days REVISION PLAN for Prelims 2020 - InstaTests

www.insightsonindia.com 31 Insights IAS

31. Consider the following statements regarding eastern coastal plain of India

1. The eastern coastal plain is broader and is an example of an emergent

coast.

2. The eastern coastal plain provides natural conditions for the

development of ports and harbours.

Which of the statements given above is/are correct?

(a) 1 only

(b) 2 only

(c) Both 1 and 2

(d) Neither 1 nor 2

Solution: A

Page 34: SIMPLYFYING IAS EXAM PREPARATION - INSIGHTSIAS · 2020. 4. 7. · Insta 75 Days Revision Plan for UPSC Civil Services Prelims – 2020 This document is the compilation of 100 questions

INSTA 75 Days REVISION PLAN for Prelims 2020 - InstaTests

www.insightsonindia.com 32 Insights IAS

• The western coastal plains are an example of submerged coastal plain. It

is believed that the city of Dwaraka which was once a part of the Indian

mainland situated along the west coast is submerged under water. Because of

this submergence it is a narrow belt and provides natural conditions for

the development of ports and harbours.

• As compared to the western coastal plain, the eastern coastal plain is

broader and is an example of an emergent coast. There are well developed

deltas here, formed by the rivers flowing eastward in to the Bay of Bengal.

These include the deltas of the Mahanadi, the Godavari, the Krishna and the

Kaveri. Because of its emergent nature, it has less number of ports and

harbours. The continental shelf extends up to 500 km into the sea, which makes

it difficult for the development of good ports and harbours.

32. 11-degree channel separates

(a) Lakshadweep and Maldives

(b) Minicoy and rest of Lakshadweep

(c) Amini Island and the Canannore Island

(d) None of the above

Solution: C

• The islands of the Arabian sea include Lakshadweep and Minicoy. These are

scattered between 8°N-12°N and 71°E -74°E longitude. The entire island group

is built of coral deposits. There are approximately 36 islands of which 11 are

inhabited.

• The entire group of islands is broadly divided by the Elevene-degree channel,

north of which is the Amini Island and to the south of the Canannore Island.

• The Islands of this archipelago have storm beaches consisting of

unconsolidated pebbles, shingles, cobbles and boulders on the eastern

seaboard.

33. Which of the following is/are the right bank tributaries of river Indus?

1. Kabul river

2. Khurram

3. Tochi

4. Gomal

Which of the statements given above is/are correct?

(a) 1, 2 and 3 only

(b) 2, 3 and 4 only

Page 35: SIMPLYFYING IAS EXAM PREPARATION - INSIGHTSIAS · 2020. 4. 7. · Insta 75 Days Revision Plan for UPSC Civil Services Prelims – 2020 This document is the compilation of 100 questions

INSTA 75 Days REVISION PLAN for Prelims 2020 - InstaTests

www.insightsonindia.com 33 Insights IAS

(c) 1, 3 and 4 only

(d) 1, 2, 3 and 4

Solution: D

The Indus receives a number of Himalayan tributaries such as the Shyok, the Gilgit,

the Zaskar, the Hunza, the Nubra, the Shigar, the Gasting and the Dras.

It finally emerges out of the hills near Attock where it receives the Kabul river on its

right bank. The other important tributaries joining the right bank of the Indus are the

Khurram, the Tochi, the Gomal, the Viboa and the Sangar. They all originate in the

Sulaiman ranges.

34. Consider the following statements regarding social forestry

1. Agro-forestry is the raising of trees and agriculture crops on the same

land inclusive of the waste patches.

2. Community forestry involves the raising of trees on public or community

land such as the village pasture.

Which of the statements given above is/are correct?

(a) 1 only

(b) 2 only

(c) Both 1 and 2

(d) Neither 1 nor 2

Solution: C

• Rural forestry lays emphasis on promotion of agro-forestry and community-

forestry.

• Agro-forestry is the raising of trees and agriculture crops on the same land

inclusive of the waste patches.

• It combines forestry with agriculture, thus, altering the simultaneous production

of food, fodder, fuel, timber and fruit.

• Community forestry involves the raising of trees on public or community land

such as the village pasture and temple land, roadside, canal bank, strips along

railway lines, and schools etc.

• Farm forestry is a term applied to the process under which farmers grow trees

for commercial and non-commercial purposes on their farm lands.

Page 36: SIMPLYFYING IAS EXAM PREPARATION - INSIGHTSIAS · 2020. 4. 7. · Insta 75 Days Revision Plan for UPSC Civil Services Prelims – 2020 This document is the compilation of 100 questions

INSTA 75 Days REVISION PLAN for Prelims 2020 - InstaTests

www.insightsonindia.com 34 Insights IAS

35. Consider the following statements regarding Elephant Endotheliotropic

Herpesvirus (EEHV)

1. It is a rare viral disease that causes fatal disease in young Asian

Elephants.

2. It is lethal for young elephants between the ages of 1 and 12.

Which of the statements given above is/are correct?

(a) 1 only

(b) 2 only

(c) Both 1 and 2

(d) Neither 1 nor 2

Solution: C

Elephant Endotheliotropic Herpesvirus (EEHV)

• Elephant Endotheliotropic Herpesvirus (EEHV) is a rare viral disease that

causes fatal disease in young Asian Elephants.

• When it is triggered, the elephant dies of massive internal bleeding and

symptoms which are hardly visible.

• Some elephants show symptoms such as reduced appetite, nasal discharge

and swollen glands.

• The disease is usually fatal, with a short course of 28-35 hours.

• It is lethal for young elephants between the ages of 1 and 12.

• There is no true cure for herpesviruses in animals or in humans.

• If a young elephant dies before reproducing, it affects the population of the

species in the concerned geography.

• It has killed five elephants in Nandan Kanan Zoo & Chandaka forest in

Odisha.

36. Which of the following are east flowing small rivers?

1. Penner

2. Palar

3. Periyar

4. Kalinadi

Select the correct answer using the code given below

(a) 1 and 2 only

(b) 2 and 3 only

(c) 2, 3 and 4 only

(d) 1, 2 and 4 only

Page 37: SIMPLYFYING IAS EXAM PREPARATION - INSIGHTSIAS · 2020. 4. 7. · Insta 75 Days Revision Plan for UPSC Civil Services Prelims – 2020 This document is the compilation of 100 questions

INSTA 75 Days REVISION PLAN for Prelims 2020 - InstaTests

www.insightsonindia.com 35 Insights IAS

Solution: A

37. Consider the following statements regarding western cyclonic disturbances

1. They enter the Indian subcontinent during the winter months and are

brought by the westerly jet stream.

2. A decrease in the prevailing night temperature generally indicates an

advance in the arrival of these cyclones’ disturbances.

3. It is highly beneficial for kharif crops.

Which of the statements given above is/are correct?

(a) 1 only

(b) 2 and 3 only

(c) 1 and 2 only

(d) 1, 2 and 3

Solution: A

The western cyclonic disturbances which enter the Indian subcontinent from the

west and the northwest during the winter months, originate over the Mediterranean

Sea and are brought into India by the westerly jet stream.

Page 38: SIMPLYFYING IAS EXAM PREPARATION - INSIGHTSIAS · 2020. 4. 7. · Insta 75 Days Revision Plan for UPSC Civil Services Prelims – 2020 This document is the compilation of 100 questions

INSTA 75 Days REVISION PLAN for Prelims 2020 - InstaTests

www.insightsonindia.com 36 Insights IAS

An increase in the prevailing night temperature generally indicates an advance in the

arrival of these cyclones’ disturbances.

In northwestern India, some weak temperate cyclones from the Mediterranean Sea

cause rainfall in Punjab, Haryana, Delhi and western Uttar Pradesh. Although the

amount is meagre, it is highly beneficial for rabi crops.

38. El-Nino results in:

1. The distortion of equatorial atmospheric circulation

2. Increase in the amount of planktons

3. Irregularities in the evaporation of sea water

Which of the statements given above is/are correct?

(a) 1 and 2 only

(b) 2 and 3 only

(c) 1 and 3 only

(d) 1, 2 and 3

Solution: C

El-Nino is a complex weather system that appears once every three to seven years,

bringing drought, floods and other weather extremes to different parts of the world.

• The system involves oceanic and atmospheric phenomena with the

appearance of warm currents off the coast of Peru in the Eastern Pacific and

affects weather in many places including India.

• El-Nino is merely an extension of the warm equatorial current which gets

replaced temporarily by cold Peruvian current or Humbolt current (locate these

currents in your atlas). This current increases the temperature of water on the

Peruvian coast by 10°C.

This results in:

(i) the distortion of equatorial atmospheric circulation;

(ii) irregularities in the evaporation of sea water;

(iii) reduction in the amount of planktons which further reduces the number of

fish in the sea.

39. Consider the following statements

1. Small Farmer’s Agri-Business Consortium (SFAC) is an autonomous

society promoted by Ministry of Commerce and Industry.

2. SFAC is a Non-Banking Financial Company.

Page 39: SIMPLYFYING IAS EXAM PREPARATION - INSIGHTSIAS · 2020. 4. 7. · Insta 75 Days Revision Plan for UPSC Civil Services Prelims – 2020 This document is the compilation of 100 questions

INSTA 75 Days REVISION PLAN for Prelims 2020 - InstaTests

www.insightsonindia.com 37 Insights IAS

Which of the statements given above is/are correct?

(a) 1 only

(b) 2 only

(c) Both 1 and 2

(d) Neither 1 nor 2

Solution: B

SFAC is an Autonomous Society promoted by Ministry of Agriculture,

Cooperation and Farmers’ Welfare, Government of India.

• It is registered under Societies Registration Act 1860. The Society is also

registered as Non-Banking Financial Institution by Reserve Bank of India.

• The role of State SFACs is to aggressively promote agribusiness project

development in their respective States.

40. Consider the following statements regarding Red soil

1. Red soil develops on crystalline igneous rocks in areas of high rainfall

2. They are generally rich in nitrogen, phosphorous and humus.

Which of the statements given above is/are correct?

(a) 1 only

(b) 2 only

(c) Both 1 and 2

(d) Neither 1 nor 2

Solution: D

Red soil develops on crystalline igneous rocks in areas of low rainfall in the eastern

and southern part of the Deccan Plateau. Along the piedmont zone of the Western

Ghat, long stretch of area is occupied by red loamy soil. Yellow and red soils are also

found in parts of Odisha and Chhattisgarh and in the southern parts of the middle

Ganga plain.

• The soil develops a reddish colour due to a wide diffusion of iron in crystalline

and metamorphic rocks. It looks yellow when it occurs in a hydrated form.

• The fine-grained red and yellow soils are normally fertile, whereas coarse-

grained soils found in dry upland areas are poor in fertility. They are generally

poor in nitrogen, phosphorous and humus.

Page 40: SIMPLYFYING IAS EXAM PREPARATION - INSIGHTSIAS · 2020. 4. 7. · Insta 75 Days Revision Plan for UPSC Civil Services Prelims – 2020 This document is the compilation of 100 questions

INSTA 75 Days REVISION PLAN for Prelims 2020 - InstaTests

www.insightsonindia.com 38 Insights IAS

41. Consider the following statements regarding local winds in northwest India

1. Loo are dry and hot winds.

2. Dust storms bring a welcome respite from the oppressing heat

Which of the statements given above is/are correct?

(a) 1 only

(b) 2 only

(c) Both 1 and 2

(d) Neither 1 nor 2

Solution: C

• In the heart of the ITCZ in the northwest, the dry and hot winds known as ‘Loo’,

blow in the afternoon, and very often, they continue to well into midnight.

• Dust storms in the evening are very common during May in Punjab, Haryana,

Eastern Rajasthan and Uttar Pradesh.

• These temporary storms bring a welcome respite from the oppressing heat

since they bring with them light rains and a pleasant cool breeze.

42. Consider the following statements regarding MANI App

1. It is launched by RBI

2. It aims to provide digital education for rural women.

3. The app authenticates a note as being either genuine or counterfeit.

Which of the statements given above is/are correct?

(a) 1 and 2 only

(b) 2 and 3 only

(c) 1 only

(d) 1, 2 and 3

Solution: C

MANI App:

• Reserve Bank of India, RBI has launched a mobile app, MANI, Mobile Aided

Note Identifier.

• It aims to help visually challenged people to identify denomination of currency

notes.

• Through the downloaded app, users can scan the notes using the camera.

• The audio output will give the result in Hindi and English.

Page 41: SIMPLYFYING IAS EXAM PREPARATION - INSIGHTSIAS · 2020. 4. 7. · Insta 75 Days Revision Plan for UPSC Civil Services Prelims – 2020 This document is the compilation of 100 questions

INSTA 75 Days REVISION PLAN for Prelims 2020 - InstaTests

www.insightsonindia.com 39 Insights IAS

• The mobile application does not authenticate a note as being either genuine or

counterfeit.

43. The Accessible India Campaign has been launched by

(a) Ministry of Women and Child Development

(b) Ministry of Human Resource Development

(c) Ministry of Health and Family Welfare

(d) Ministry of Social Justice and Empowerment

Solution: D

The Department of Empowerment of Persons with Disabilities (DEPwD) under

Ministry of Social Justice and Empowerment has developed a Management

Information System (MIS) for stakeholders of Accessible India Campaign (AIC).

• The portal will bring all the nodal ministries, and States/UTs on a single platform

for monitoring the progress being made against each target of AIC.

About Accessible India Campaign:

• Accessible India Campaign (AIC) is the nationwide flagship campaign of the

Department of Empowerment of Persons with Disabilities (DEPwD), Ministry of

Social Justice and Empowerment.

• Aim: The aim of the Campaign is to make a barrier free and conducive

environment for Divyangjans all over the country. The campaign has the vision

to build an inclusive society in which equal opportunities are provided for the

growth and development of Persons with Disabilities (PwDs) so that they can

lead productive, safe and dignified lives.

• Implementation: For creating universal accessibility for Persons with

Disabilities, the campaign has been divided into three verticals: Built

Environment; Transport and Information & Communication Technology (ICT)

ecosystem.

Targets:

• Completing accessibility audit of at least 25-50 most important government

buildings in 50 cities and making them fully accessible by the end of this year.

• Making 50% of all the government buildings of NCT and all the State capitals

fully accessible by December 2018.

• Completing accessibility audit of 50% of government buildings and making

them fully accessible in 10 most important cities/towns of States not covered in

targets (i) and (ii) by December 2019.

Page 42: SIMPLYFYING IAS EXAM PREPARATION - INSIGHTSIAS · 2020. 4. 7. · Insta 75 Days Revision Plan for UPSC Civil Services Prelims – 2020 This document is the compilation of 100 questions

INSTA 75 Days REVISION PLAN for Prelims 2020 - InstaTests

www.insightsonindia.com 40 Insights IAS

• Accessible India Campaign is in line with the Article 9 of UNCRPD (UN

Convention on the Rights of Persons with Disabilities) to which India is a

signatory since 2007.

https://www.insightsonindia.com/2019/09/19/accessible-india-campaign/

44. According to Koeppen’s scheme, Monsoon with dry season in summer can be

found in which parts of India

(a) Northeastern India

(b) Central India

(c) South east India

(d) South west India

Solution: C

Page 43: SIMPLYFYING IAS EXAM PREPARATION - INSIGHTSIAS · 2020. 4. 7. · Insta 75 Days Revision Plan for UPSC Civil Services Prelims – 2020 This document is the compilation of 100 questions

INSTA 75 Days REVISION PLAN for Prelims 2020 - InstaTests

www.insightsonindia.com 41 Insights IAS

45. Consider the following statements regarding The High-Biodiversity Wilderness

Areas (HBWA)

1. HBWA approach has been developed by IUCN.

2. Wilderness areas are classified as areas ≥10,000 km2 that are based on

the world’s terrestrial ecoregions.

3. India has no High-Biodiversity Wilderness Areas

Which of the statements given above is/are correct?

(a) 2 only

(b) 3 only

(c) 2 and 3 only

(d) 1 and 2 only

Solution: C

The High Biodiversity Wilderness Areas (HBWA) approach has been developed

by Conservation International (CI). HBWAs consist of 5 of the 24 major wilderness

areas that hold globally significant levels of biodiversity, as identified by Mittermeier et

al (2002).

• The 5 HBWAs are Amazonia, the Congo forests of Central Africa, New

Guinea, the Miombo-Mopane woodlands of Southern Africa (including the

Okavango Delta), and the North American desert complex of northern

Mexico and the Southwestern part of United States of America.

• The intact portion of these areas covers 8,981,000 km2 (76% of their original

extent), and 6.1% of the planet’s land area. 2 The geographic boundaries of the

HBWAs coincide with the boundaries of several amalgamated WWF

ecoregions.

Criteria:

• Wilderness areas are classified as areas ≥10,000 km2 that are based on the

world’s terrestrial ecoregions, with a human population density outside urban

areas of ≤5 people per km2 that retain at least 70% of their historical habitat

extent (500 years ago). This analysis yielded 24 wilderness areas. 2 The HBWA

are the top five wilderness areas based on endemic biodiversity (at least 0.5%

of the world’s plants).

India has no High-Biodiversity Wilderness Areas

http://www.biodiversitya-z.org/content/high-biodiversity-wilderness-areas-hbwa

Page 44: SIMPLYFYING IAS EXAM PREPARATION - INSIGHTSIAS · 2020. 4. 7. · Insta 75 Days Revision Plan for UPSC Civil Services Prelims – 2020 This document is the compilation of 100 questions

INSTA 75 Days REVISION PLAN for Prelims 2020 - InstaTests

www.insightsonindia.com 42 Insights IAS

46. Consider the following statements regarding The United Nations Relief and

Works Agency

1. It is a relief and human development agency

2. It is the only UN agency dedicated to helping refugees from a specific

region or conflict

3. It is funded almost entirely by voluntary contributions from UN Member

States.

Which of the statements given above is/are not correct?

(a) 1, 2 and 3

(b) 2 and 3 only

(c) 3 only

(d) None

Solution: D

India has contributed USD 5 million in 2019 to UN Palestine refugee agency. The

contribution was provided in support of UNRWA’s core programmes and services,

including education, health care, and relief and social services.

Background:

• India has increased its annual financial contribution fourfold to the United

Nations Relief and Works Agency for Palestine Refugees in the Near East

(UNRWA) core budget, from USD 1.25 million in 2016 to USD 5 million in 2018.

UNRWA:

• It is a relief and human development agency that was established in 8

December 1949.

• Following the 1948 Arab-Israeli conflict, UNRWA was established by United

Nations General Assembly to carry out direct relief and works programmes for

Palestine refugees. The Agency began operations on 1 May 1950.

• UNRWA is the only UN agency dedicated to helping refugees from a specific

region or conflict and is separate from UNHCR.

• Funding: UNRWA is funded almost entirely by voluntary contributions from UN

Member States. UNRWA also receives some funding from the Regular Budget

of the United Nations, which is used mostly for international staffing costs.

• Aid is provided in five areas of operation: Jordan, Lebanon, Syria, the Gaza

Strip and the West Bank, including East Jerusalem; aid for Palestinian

refugees outside these five areas is provided by UNHCR.

Functions:

• UNRWA has been providing health, education, relief and social services, as

well as emergency humanitarian assistance, across its five fields of operation

Jordan, Lebanon, Syria, West Bank and the Gaza Strip since 1950.

Page 45: SIMPLYFYING IAS EXAM PREPARATION - INSIGHTSIAS · 2020. 4. 7. · Insta 75 Days Revision Plan for UPSC Civil Services Prelims – 2020 This document is the compilation of 100 questions

INSTA 75 Days REVISION PLAN for Prelims 2020 - InstaTests

www.insightsonindia.com 43 Insights IAS

• The Agency currently serves 5.4 million Palestinian refugees 20% of the world’s

refugees.

Challenges before UNRWA:

• The UN agency is going through a difficult financial situation due to voluntary

contributions from a limited donor base.

• There is a projected shortfall of more than USD 200 million against a funding

requirement of approximately USD 1.2 billion for UNRWA this year.

• The shortfall may impact the agency’s ability to provide essential services to

the Palestine refugees, notably in the fields of education, health, and assistance

to the most vulnerable refugees.

• Besides, a corruption scandal involving sexual misconduct, nepotism,

retaliation against whistleblowers and lots of business-class travel has gripped

the United Nations Relief and Works Agency for Palestine Refugees in the Near

East.

https://www.insightsonindia.com/2019/08/08/un-palestine-refugee-agency-2/

47. Consider the following statements regarding ‘One Nation One Ration Card’

scheme

1. It aims to provide portability of food security benefits all across the nation.

2. It is launched by Ministry of Consumer Affairs and Food and Public

Distribution.

3. It would integrate the existing PDS systems/portals of States/UTs with

the Central systems/portals.

Which of the statements given above is/are correct?

(a) 1 and 3 only

(b) 2 and 3 only

(c) 1 and 2 only

(d) 1, 2 and 3

Solution: D

• Government has launched One Nation-One Ration Card scheme on pilot

basis in Telangana, Andhra Pradesh, Maharastra and Gujarat. Families who

have food security cards can buy subsidized rice and wheat from any ration

shop in these states but their ration cards should be linked with Aadhar Number

to avail this service.

Page 46: SIMPLYFYING IAS EXAM PREPARATION - INSIGHTSIAS · 2020. 4. 7. · Insta 75 Days Revision Plan for UPSC Civil Services Prelims – 2020 This document is the compilation of 100 questions

INSTA 75 Days REVISION PLAN for Prelims 2020 - InstaTests

www.insightsonindia.com 44 Insights IAS

About the scheme:

• One Nation One Ration Card (RC) will ensure all beneficiaries especially

migrants can access PDS across the nation from any PDS shop of their own

choice.

• Benefits: no poor person is deprived of getting subsidized food grains under

the food security scheme when they shift from one place to another. It also aims

to remove the chance of anyone holding more than one ration card to avail

benefits from different states.

• Significance: This will provide freedom to the beneficiaries as they will not be

tied to any one PDS shop and reduce their dependence on shop owners and

curtail instances of corruption.

• It is launched by Ministry of Consumer Affairs, Food and Public

Distribution.

• It would integrate the existing PDS systems/portals of States/UTs with the

Central systems/portals.

Challenges:

• Prone to corruption: Every state has its own rules for Public Distribution

System (PDS). If ‘One Nation, One Ration Card’ is implemented, it will further

boost corruption in an already corrupted Public Distribution System.

• The scheme will increase the woes of the common man and, the middlemen

and corrupt PDS shop owners will exploit them.

• Tamil Nadu has opposed the proposal of the Centre, saying it would result in

undesirable consequences and is against federalism.

https://www.insightsonindia.com/2019/08/02/one-nation-one-ration-card-2/

48. Consider the following statements regarding Convention on International Trade

in Endangered Species of Wild Fauna and Flora (CITES)

1. It is an international agreement to regulate worldwide commercial trade

in wild animal and plant species

2. It is administered by the World Animal Foundation.

3. It is legally binding on the Parties, but it does not take the place of

national laws.

Which of the statements given above is/are correct?

(a) 1 only

(b) 2 and 3 only

(c) 1 and 3 only

(d) 1, 2 and 3

Solution: C

Page 47: SIMPLYFYING IAS EXAM PREPARATION - INSIGHTSIAS · 2020. 4. 7. · Insta 75 Days Revision Plan for UPSC Civil Services Prelims – 2020 This document is the compilation of 100 questions

INSTA 75 Days REVISION PLAN for Prelims 2020 - InstaTests

www.insightsonindia.com 45 Insights IAS

Convention on International Trade in Endangered Species of Wild Fauna and

Flora (CITES):

• It is an International agreement to regulate worldwide commercial trade in wild

animal and plant species.

• It restricts trade in items made from such plants and animals, such as food,

clothing, medicine, and souvenirs.

• It was signed on March 3, 1973 (Hence world wildlife day is celebrated on

march 3).

• It is administered by the United Nations Environment Programme (UNEP).

• Secretariat— Geneva (Switzerland).

• It is legally binding on the Parties, but it does not take the place of national laws.

Classifications:

It classifies plants and animals according to three categories, or appendices, based

on how threatened. They are.

• Appendix I: It lists species that are in danger of extinction. It prohibits

commercial trade of these plants and animals except in extraordinary situations

for scientific or educational reasons.

• Appendix II species: They are those that are not threatened with extinction but

that might suffer a serious decline in number if trade is not restricted. Their trade

is regulated by permit.

• Appendix III species: They are protected in at least one country that is a

CITES member states and that has petitioned others for help in controlling

international trade in that species.

https://www.insightsonindia.com/2019/08/22/cites-washington-convention-3/

49. Consider the following statements regarding Dadabhai Naoroji

1. He was the first Indian member of the British parliament.

2. He was Congress president for five times.

Which of the statements given above is/are correct?

(a) 1 only

(b) 2 only

(c) Both 1 and 2

(d) Neither 1 nor 2

Solution: A

September 4, 2019 was the 194th birth anniversary of Dadabhai Naoroji, the “Grand

Old Man of India”, who was among the first leaders who stirred national

consciousness in the country. Born in 1825 at Navsari, in present-day Gujarat.

Page 48: SIMPLYFYING IAS EXAM PREPARATION - INSIGHTSIAS · 2020. 4. 7. · Insta 75 Days Revision Plan for UPSC Civil Services Prelims – 2020 This document is the compilation of 100 questions

INSTA 75 Days REVISION PLAN for Prelims 2020 - InstaTests

www.insightsonindia.com 46 Insights IAS

Key contributions:

• He was closely involved with the Indian National Congress in its early phase.

• He served as the first Indian member of the British parliament.

• His first agitation, in 1859, concerned recruitment to the Indian Civil Service.

• In 1865 and 1866, Naoroji helped found the London Indian Society and the

East India Association The two organisations sought to bring nationalist

Indians and sympathetic Britons on one platform.

• As the secretary of the East India Association, Naoroji travelled in India to

gather funds and raise national awareness.

• In 1885, Naoroji became a vice-president of the Bombay Presidency

Association, was nominated to the Bombay legislative council by Governor

Lord Reay, and helped form the Indian National Congress.

• He was Congress president thrice, in 1886, 1893, and 1906.

• In 1893, he helped form an Indian parliamentary committee to attend to

Indian interests.

• In 1895, he became a member of the royal commission on Indian

expenditure.

• Dadabhai Naoroji was among the key proponents of the ‘Drain Theory’,

disseminating it in his 1901 book ‘Poverty and Un-British Rule in India’.

https://www.insightsonindia.com/2019/09/05/dadabhai-naoroji/

50. Sholas can be found in which of the following hills?

1. Nilgiris

2. Anaimalai hills

3. Palani hills

Select the correct answer using the code given below

(a) 1 only

(b) 2 and 3 only

(c) 1 and 2 only

(d) 1, 2 and 3

Solution: D

The temperate forests are called Sholas in the Nilgiris, Anaimalai and Palani hills.

Some of the other trees of this forest of economic significance include, magnolia,

laurel, cinchona and wattle. Such forests are also found in the Satpura and the

Maikal ranges.

Page 49: SIMPLYFYING IAS EXAM PREPARATION - INSIGHTSIAS · 2020. 4. 7. · Insta 75 Days Revision Plan for UPSC Civil Services Prelims – 2020 This document is the compilation of 100 questions

INSTA 75 Days REVISION PLAN for Prelims 2020 - InstaTests

www.insightsonindia.com 47 Insights IAS

DAY – 15 (InstaTest-15)

51. Consider the following statements regarding South Asia Satellite

1. It is an earth observation satellite launched by ISRO

2. It offers the services in different areas like tele-medicine, disaster

management and banking.

Which of the statements given above is/are correct?

(a) 1 only

(b) 2 only

(c) Both 1 and 2

(d) Neither 1 nor 2

Solution: B

South Asia Satellite GSAT-9 is a Geostationary Communication satellite

launched by India. The primary objective of GSAT-9 is to provide various

communication applications in Ku-band with coverage over South Asian countries.

• India’s Geosynchronous Satellite Launch Vehicle (GSLV-F09) successfully

launched the 2230 kg South |Asia Satellite (GSAT-9) into its planned

Geosynchronous Transfer Orbit (GTO).

• The two solar arrays of GSAT-9 consisting of Ultra Triple Junction solar cells

generate about 3500 Watts of electrical power. Sun and Earth sensors as well

as gyroscopes provide orientation reference for the satellite. The Attitude and

Orbit Control System (AOCS) of the satellite maintains it’s orientation with the

help of momentum wheels, magnetic torquers and thrusters.

• The satellite’s propulsion system consists of a Liquid Apogee Motor (LAM) and

chemical thrusters using liquid propellants for initial orbit raising and station

keeping. The satellite also carries plasma thrusters, assisting in station keeping.

• It offers the services in different areas like tele-medicine, disaster management

and banking.

• India will set up five ground stations and more than 500 terminals in five

neighboring countries – Bhutan, Nepal, Maldives, Bangladesh and Sri Lanka.

• Pakistan did not join the project, stating that it was working on its own satellite,

but did offer “monetary and technical support”. India rejected Pakistani offers,

saying that it wanted the project to be a “gift” and multi-national collaboration

would be time consuming.

https://www.isro.gov.in/update/05-may-2017/gslv-successfully-launches-south-asia-

satellite

Page 50: SIMPLYFYING IAS EXAM PREPARATION - INSIGHTSIAS · 2020. 4. 7. · Insta 75 Days Revision Plan for UPSC Civil Services Prelims – 2020 This document is the compilation of 100 questions

INSTA 75 Days REVISION PLAN for Prelims 2020 - InstaTests

www.insightsonindia.com 48 Insights IAS

52. Which of the following rivers is/are tributaries of River Krishna

1. Manjra

2. Koyna

3. Tungbhadra

4. Bhima

Select the correct answer using the code given below:

(a) 1, 2 and 3 only

(b) 2, 3 and 4 only

(c) 1, 3 and 4 only

(d) 1, 2, 3 and 4

Solution: B

The Penganga, the Indravati, the Pranhita, and the Manjra are Godavari’s principal

tributaries.

The Krishna is the second largest eastflowing Peninsular river which rises near

Mahabaleshwar in Sahyadri. Its total length is 1,401 km. The Koyna, the Tungbhadra

and the Bhima are its major tributaries.

Extra Learning :

About River Krishna:

Of the total catchment area of the Krishna, 27 per cent lies in Maharashtra, 44 per

cent in Karnataka and 29 per cent in Andhra Pradesh and Telangana.

Page 51: SIMPLYFYING IAS EXAM PREPARATION - INSIGHTSIAS · 2020. 4. 7. · Insta 75 Days Revision Plan for UPSC Civil Services Prelims – 2020 This document is the compilation of 100 questions

INSTA 75 Days REVISION PLAN for Prelims 2020 - InstaTests

www.insightsonindia.com 49 Insights IAS

Important Dams on River Krishna:

1. Almatti Dam

2. Basava Sagar Dam

3. Nagarjuna Sagar Dam

4. Srisailam Dam

5. Jurala Dam

6. Narayanpur Dam (downriver of Almatti Dam)

7. Pulichitnthala Dam is under construction.

8. Amar Dam

Other information:

• There are famous religious places such as Narsobawadi and Audumber on

the riverbanks close to Sangli.

• The riverbanks are also famous for the International Kalachakra Festival,

which was observed in front of the Dalai Lama in Amaravati. Amaravati served

as the capital of the Satavahana kings who ruled South India for 400 years and

a significant place for studying and gathering insights on Buddhism.

53. Which of the following are the reasons for dry season in Tamil Nadu during

monsoons

1. The Tamil Nadu coast is situated parallel to the Bay of Bengal branch of

southwest monsoon.

2. It lies in the rain shadow area of the Arabian Sea branch of the south-

west monsoon.

Which of the statement above is/are correct?

(a) 1 only

(b) 2 only

(c) Both 1 and 2

(d) Neither 1 nor 2

Solution: C

Why does Tamil Nadu coast remains dry during monsoon season. There are two

factors responsible for it:

(i) The Tamil Nadu coast is situated parallel to the Bay of Bengal branch

of southwest monsoon.

(ii) It lies in the rain shadow area of the Arabian Sea branch of the south-

west monsoon.

Page 52: SIMPLYFYING IAS EXAM PREPARATION - INSIGHTSIAS · 2020. 4. 7. · Insta 75 Days Revision Plan for UPSC Civil Services Prelims – 2020 This document is the compilation of 100 questions

INSTA 75 Days REVISION PLAN for Prelims 2020 - InstaTests

www.insightsonindia.com 50 Insights IAS

Extra Learning:

54. Consider the following statements regarding contesting an election in India

1. A person is who not registered as a voter in any of the constituency in

India can also contest in election.

2. A person confined in jail can also vote in an election.

Which of the statements given above is/are correct?

(a) 1 only

(b) 2 only

(c) Both 1 and 2

(d) Neither 1 nor 2

Solution: D

If I am not registered as a voter in any Constituency, can I contest election?

Ans. No

Page 53: SIMPLYFYING IAS EXAM PREPARATION - INSIGHTSIAS · 2020. 4. 7. · Insta 75 Days Revision Plan for UPSC Civil Services Prelims – 2020 This document is the compilation of 100 questions

INSTA 75 Days REVISION PLAN for Prelims 2020 - InstaTests

www.insightsonindia.com 51 Insights IAS

• For contesting an election as a candidate a person must be registered as a

voter. Sec 4 (d) of Representation People Act, 1951 precludes a person from

contesting unless he is an elector in any parliamentary constituency. Section 5

(c) of R. P. Act, 1951 has a similar provision for Assembly Constituencies.

• If you are a registered voter in Delhi, you can contest an election to Lok Sabha

from any constituency in the country except Assam, Lakshadweep and Sikkim,

as per Section 4 (c), 4 (cc) and 4 (ccc) of the R. P. Act, 1951.

Can a person confined in jail vote in an election?

Ans. No

• According to section 62(5) of the Representation of the People Act, 1951, no

person shall vote at any election if he is confined in a prison, whether under a

sentence of imprisonment or transportation or otherwise, or is in the lawful

custody of the police.

https://eci.gov.in/faqs/elections/contesting-for-elections/faqs-contesting-for-elections-

r4/

55. Which of the following is/are considered as the pillars of Human development

1. Equity

2. Sustainability

3. Human Labour Productivity

4. Empowerment

Select the correct answer using the code given below:

(a) 1 and 4 only

(b) 1, 2 and 4 only

(c) 1, 2, 3 and 4

(d) None

Solution: C

The Four Pillars of Human Development:

• Just as any building is supported by pillars, the idea of human development is

supported by the concepts of equity, sustainability, productivity and

empowerment.

Page 54: SIMPLYFYING IAS EXAM PREPARATION - INSIGHTSIAS · 2020. 4. 7. · Insta 75 Days Revision Plan for UPSC Civil Services Prelims – 2020 This document is the compilation of 100 questions

INSTA 75 Days REVISION PLAN for Prelims 2020 - InstaTests

www.insightsonindia.com 52 Insights IAS

Extra Learning:

Equity refers to making equal access to opportunities available to everybody. The

opportunities available to people must be equal irrespective of their gender, race,

income and in the Indian case, caste. Yet this is very often not the case and happens

in almost every society. For example, in any country, it is interesting to see which

group the most of the school dropouts belong to. This should then lead to an

understanding of the reasons for such behaviour. In India, a large number of women

and persons belonging to socially and economically backward groups drop out of

school. This shows how the choices of these groups get limited by not having access

to knowledge.

Sustainability means continuity in the availability of opportunities. To have

sustainable human development, each generation must have the same opportunities.

All environmental, financial and human resources must be used keeping in mind the

future. Misuse of any of these resources will lead to fewer opportunities for future

generations. A good example is about the importance of sending girls to school. If a

community does not stress the importance of sending its girl children to school, many

opportunities will be lost to these young women when they grow up. Their career

choices will be severely curtailed and this would affect other aspects of their lives. So

each generation must ensure the availability of choices and opportunities to its future

generations.

Productivity here means human labour productivity or productivity in terms of human

work. Such productivity must be constantly enriched by building capabilities in people.

Ultimately, it is people who are the real wealth of nations. Therefore, efforts to increase

their knowledge, or provide better health facilities ultimately leads to better work

efficiency.

Empowerment means to have the power to make choices. Such power comes from

increasing freedom and capability. Good governance and people-oriented policies are

required to empower people. The empowerment of socially and economically

disadvantaged groups is of special importance.

56. Consider the following statements regarding Measuring of Earthquakes

1. The Mercalli scale measures the intensity of the earthquake shock.

2. The intensity scale takes into account the visible damage caused by the

event.

3. The range of intensity scale is from 0-10.

Which of the statements given above is/are correct?

(a) 1 and 2 only

(b) 2 and 3 only

(c) 1 and 3 only

(d) 1, 2 and 3

Page 55: SIMPLYFYING IAS EXAM PREPARATION - INSIGHTSIAS · 2020. 4. 7. · Insta 75 Days Revision Plan for UPSC Civil Services Prelims – 2020 This document is the compilation of 100 questions

INSTA 75 Days REVISION PLAN for Prelims 2020 - InstaTests

www.insightsonindia.com 53 Insights IAS

Solution: A

Measuring Earthquakes

The earthquake events are scaled either according to the magnitude or intensity of the

shock. The magnitude scale is known as the Richter scale. The magnitude relates to

the energy released during the quake. The magnitude is expressed in numbers, 0-10.

The Richter magnitude scale is a scale of numbers used to tell the power (or

magnitude) of earthquakes. Charles Richter developed the Richter Scale in 1935. His

scale worked like a seismogram, measured by a particular type of seismometer at a

distance of 100 kilometers (62 mi) from the earthquake.

The intensity scale is named after Mercalli, an Italian seismologist. The intensity scale

takes into account the visible damage caused by the event. The range of intensity

scale is from 1-12.

Extra Learning:

EFFECTS OF EARTHQUAKE:

Earthquake is a natural hazard. The following are the immediate hazardous effects of

earthquake:

(i) Ground Shaking

(ii) Differential ground settlement

(iii) Land and mud slides

(iv) Soil liquefaction

(v) Ground lurching

(vi) Avalanches

(vii) Ground displacement

(viii) Floods from dam and levee failures

(ix) Fires

(x) Structural collapse

(xi) Falling objects

(xii) Tsunami

The first six listed above have some bearings upon landforms, while others may be

considered the effects causing immediate concern to the life and properties of people

in the region.

Page 56: SIMPLYFYING IAS EXAM PREPARATION - INSIGHTSIAS · 2020. 4. 7. · Insta 75 Days Revision Plan for UPSC Civil Services Prelims – 2020 This document is the compilation of 100 questions

INSTA 75 Days REVISION PLAN for Prelims 2020 - InstaTests

www.insightsonindia.com 54 Insights IAS

57. Consider the following statements regarding Ashgabat agreement

1. It is a multimodal transport agreement that envisages the creation of an

International Transport and Transit Corridor connecting Central Asia with

the Persian Gulf.

2. Pakistan, Afghanistan and India have acceded to the Ashgabat

agreement.

3. The agreement was originally signed by Iran, India and Russia in 2011.

Which of the statements given above is/are correct?

(a) 1 and 3 only

(b) 1 only

(c) 2 and 3 only

(d) 1, 2 and 3

Page 57: SIMPLYFYING IAS EXAM PREPARATION - INSIGHTSIAS · 2020. 4. 7. · Insta 75 Days Revision Plan for UPSC Civil Services Prelims – 2020 This document is the compilation of 100 questions

INSTA 75 Days REVISION PLAN for Prelims 2020 - InstaTests

www.insightsonindia.com 55 Insights IAS

Solution: B

• The Ashgabat agreement is a multimodal transport agreement between the

governments of Kazakhstan, Uzbekistan, Turkmenistan, Iran, Pakistan,

India and Oman for creating an international transport and transit corridor

facilitating transportation of goods between Central Asia and the Persian Gulf.

• The agreement was originally signed by Iran, Oman, Qatar, Turkmenistan and

Uzbekistan on April 25, 2011. Qatar subsequently withdrew from the agreement

in 2013, the same year Kazakhstan applied for membership, which was

eventually approved in 2015. Pakistan has also joined the Agreement from

November 2016. India formally joined in February 2018

• The agreement came into force in April 2016. Ashgabat in Turkmenistan is the

depository state for the agreement

• Afghanistan has not acceded the Ashagabat agreement.

https://idsa.in/idsacomments/significance-of-india-joining-the-ashgabat-

agreement_p-stobdan-120218

58. Consider the following statements regarding Plate tectonics

1. McKenzie and Parker came out with the concept of Plate Tectonics.

2. A tectonic plate is a massive, irregularly-shaped slab of solid rock,

generally composed of both continental and oceanic lithosphere.

Which of the statements given above is/are correct?

(a) 1 only

(b) 2 only

(c) Both 1 and 2

(d) Neither 1 nor 2

Solution: C

Plate tectonics

Since the advent of the concept of sea floor spreading, the interest in the problem of

distribution of oceans and continents was revived. It was in 1967, McKenzie and

Parker and also Morgan, independently collected the available ideas and came out

with another concept termed Plate Tectonics. A tectonic plate (also called lithospheric

plate) is a massive, irregularly-shaped slab of solid rock, generally composed of both

continental and oceanic lithosphere.

Plates move horizontally over the asthenosphere as rigid units. The lithosphere

includes the crust and top mantle with its thickness range varying between 5 and100

km in oceanic parts and about 200 km in the continental areas.

Page 58: SIMPLYFYING IAS EXAM PREPARATION - INSIGHTSIAS · 2020. 4. 7. · Insta 75 Days Revision Plan for UPSC Civil Services Prelims – 2020 This document is the compilation of 100 questions

INSTA 75 Days REVISION PLAN for Prelims 2020 - InstaTests

www.insightsonindia.com 56 Insights IAS

A plate may be referred to as the continental plate or oceanic plate depending on

which of the two occupy a larger portion of the plate. Pacific plate is largely an oceanic

plate whereas the Eurasian plate may be called a continental plate.

The theory of plate tectonics proposes that the earth’s lithosphere is divided into seven

major and some minor plates. Young Fold Mountain ridges, trenches, and/or faults

surround these major plates.

Extra Learning:

The major plates are as follows :

I. Antarctica and the surrounding oceanic plate

II. North American (with western Atlantic floor separated from the South American

plate along the Caribbean islands) plate

III. South American (with western Atlantic floor separated from the North American

plate along the Caribbean islands) plate

IV. Pacific plate

V. India-Australia-New Zealand plate

VI. Africa with the eastern Atlantic floor plate

VII. Eurasia and the adjacent oceanic plate.

Some important minor plates are listed below:

(i) Cocos plate : Between Central America and Pacific plate

(ii) Nazca plate : Between South America and Pacific plate

(iii) Arabian plate : Mostly the Saudi Arabian landmass

(iv) Philippine plate: Between the Asiatic and Pacific plate

Page 59: SIMPLYFYING IAS EXAM PREPARATION - INSIGHTSIAS · 2020. 4. 7. · Insta 75 Days Revision Plan for UPSC Civil Services Prelims – 2020 This document is the compilation of 100 questions

INSTA 75 Days REVISION PLAN for Prelims 2020 - InstaTests

www.insightsonindia.com 57 Insights IAS

59. Consider the following statements regarding Mixed Farming

1. Equal emphasis is laid on crop cultivation and animal husbandry.

2. Fodder crops are an important component of mixed farming.

3. Crop rotation and intercropping play an important role in maintaining soil

fertility.

Which of the statements given above is/are correct?

(a) 1 and 2 only

(b) 2 and 3 only

(c) 1 and 3 only

(d) 1, 2 and 3

Solution: D

Mixed Farming

• This form of agriculture is found in the highly developed parts of the world, e.g.

North-western Europe, Eastern North America, parts of Eurasia and the

temperate latitudes of Southern continents.

• Mixed farms are moderate in size and usually the crops associated with it are

wheat, barley, oats, rye, maize, fodder and root crops. Fodder crops are an

important component of mixed farming. Crop rotation and intercropping play an

Page 60: SIMPLYFYING IAS EXAM PREPARATION - INSIGHTSIAS · 2020. 4. 7. · Insta 75 Days Revision Plan for UPSC Civil Services Prelims – 2020 This document is the compilation of 100 questions

INSTA 75 Days REVISION PLAN for Prelims 2020 - InstaTests

www.insightsonindia.com 58 Insights IAS

important role in maintaining soil fertility. Equal emphasis is laid on crop

cultivation and animal husbandry. Animals like cattle, sheep, pigs and poultry

provide the main income along with crops.

• Mixed farming is characterized by high capital expenditure on farm machinery

and building, extensive use of chemical fertilizers and green manures and also

by the skill and expertise of the farmers.

60. Consider the following statements regarding Fall armyworm

1. It is an insect native to tropical and subtropical regions of the Americas.

2. It is an invasive species impacting the productivity of maize, rice and

sugarcane in India.

Which of the statements given above is/are correct?

(a) 1 only

(b) 2 only

(c) Both 1 and 2

(d) Neither 1 nor 2

Solution: C

Fall Armyworm (FAW), or Spodoptera frugiperda, is an insect that is native to

tropical and subtropical regions of the Americas.

In the absence of natural control or good management, it can cause significant

damage to crops. It prefers maize, but can feed on more than 80 additional species of

crops, including rice, sorghum, millet, sugarcane, vegetable crops and cotton.

http://www.fao.org/fall-armyworm/en/

61. Arrange the following cities of India from north to south

1. Agra

2. Latur

3. Gandhinagar

4. Delhi

Select the correct answer using the code given below:

(a) 4 1 3 2

(b) 4 2 3 1

(c) 1 4 2 3

(d) 1 3 2 4

Page 61: SIMPLYFYING IAS EXAM PREPARATION - INSIGHTSIAS · 2020. 4. 7. · Insta 75 Days Revision Plan for UPSC Civil Services Prelims – 2020 This document is the compilation of 100 questions

INSTA 75 Days REVISION PLAN for Prelims 2020 - InstaTests

www.insightsonindia.com 59 Insights IAS

Solution: A

62. Consider the following statements regarding Laterite Soil

1. The laterite soils develop in areas with high temperature and high rainfall.

2. These soils are poor in organic matter, nitrogen, phosphate and calcium.

3. Red laterite soils are suitable for tree crops like cashewnut.

Which of the statements given above is/are correct?

Page 62: SIMPLYFYING IAS EXAM PREPARATION - INSIGHTSIAS · 2020. 4. 7. · Insta 75 Days Revision Plan for UPSC Civil Services Prelims – 2020 This document is the compilation of 100 questions

INSTA 75 Days REVISION PLAN for Prelims 2020 - InstaTests

www.insightsonindia.com 60 Insights IAS

(a) 1 and 2 only

(b) 2 and 3 only

(c) 1 and 3 only

(d) 1, 2 and 3

Solution: D

Laterite Soil

Laterite has been derived from the Latin word ‘Later’ which means brick. The laterite

soils develop in areas with high temperature and high rainfall. These are the result of

intense leaching due to tropical rains. With rain, lime and silica are leached away, and

soils rich in iron oxide and aluminium compound are left behind. Humus content of the

soil is removed fast by bacteria that thrives well in high temperature. These soils are

poor in organic matter, nitrogen, phosphate and calcium, while iron oxide and potash

are in excess. Hence, laterites are not suitable for cultivation; however, application of

manures and fertilisers are required for making the soils fertile for cultivation.

Red laterite soils in Tamil Nadu, Andhra Pradesh and Kerala are more suitable for tree

crops like cashewnut.

Laterite soils are widely cut as bricks for use in house construction. These soils have

mainly developed in the higher areas of the Peninsular plateau. The laterite soils are

commonly found in Karnataka, Kerala, Tamil Nadu, Madhya Pradesh and the hilly

areas of Odisha and Assam.

Extra Learning:

• The National Bureau of Soil

Survey and the Land Use

Planning an Institute under

the control of the Indian

Council of Agricultural

Research (ICAR) did a lot of

studies on Indian soils.

• In their effort to study soil and

to make it comparable at the

international level, the ICAR

has classified the Indian soils

on the basis of their nature

and character as per the

United States Department of

Agriculture (USDA) Soil

Taxonomy.

Page 63: SIMPLYFYING IAS EXAM PREPARATION - INSIGHTSIAS · 2020. 4. 7. · Insta 75 Days Revision Plan for UPSC Civil Services Prelims – 2020 This document is the compilation of 100 questions

INSTA 75 Days REVISION PLAN for Prelims 2020 - InstaTests

www.insightsonindia.com 61 Insights IAS

On the basis of genesis, colour, composition and location, the soils of India have been

classified into:

(i) Alluvial soils

(ii) Black soils

(iii) Red and Yellow soils

(iv) Laterite soils

(v) Arid soils

(vi) Saline soils

(vii) Peaty soils

(viii) Forest soils

Page 64: SIMPLYFYING IAS EXAM PREPARATION - INSIGHTSIAS · 2020. 4. 7. · Insta 75 Days Revision Plan for UPSC Civil Services Prelims – 2020 This document is the compilation of 100 questions

INSTA 75 Days REVISION PLAN for Prelims 2020 - InstaTests

www.insightsonindia.com 62 Insights IAS

63. Consider the following statements regarding Irrawaddy Dolphins

1. They usually live in brackish water.

2. Bhitarkanika Wildlife Sanctuary is the natural habitat of it

3. It is a least concerned species as per IUCN status.

Which of the statements given above is/are correct?

(a) 2 only

(b) 2 and 3 only

(c) 1 and 2 only

(d) 1 only

Solution: C

• The Irrawaddy dolphin (Orcaella brevirostris) is a euryhaline species of

oceanic dolphin found in discontinuous subpopulations near sea coasts and in

estuaries and rivers in parts of the Bay of Bengal and Southeast Asia.

• It is inhabiting in the Bhitarkanika Wildlife Sanctuary and Gahirmatha

Marine Sanctuary. Its habitat range extends from the Bay of Bengal to New

Guinea and the Philippines

• It is an endangered species

• The protection of the Irrawaddy dolphin is crucial for the overall health of the

Mekong River—home to an estimated 1,100 species of fish. The Irrawaddy

dolphin is also regarded as a sacred animal by both Khmer and Lao people,

and is an important source of income and jobs for communities involved in

dolphin-watching ecotourism.

https://www.worldwildlife.org/species/irrawaddy-dolphin

64. Consider the following statements regarding Coriolis Force

1. It deflects the wind to left the direction in the northern hemisphere and to

the right in the southern hemisphere.

2. The Coriolis force is directly proportional to the angle of latitude.

Which of the statements given above is/are correct?

(a) 1 only

(b) 2 only

(c) Both 1 and 2

(d) Neither 1 nor 2

Solution: B

Page 65: SIMPLYFYING IAS EXAM PREPARATION - INSIGHTSIAS · 2020. 4. 7. · Insta 75 Days Revision Plan for UPSC Civil Services Prelims – 2020 This document is the compilation of 100 questions

INSTA 75 Days REVISION PLAN for Prelims 2020 - InstaTests

www.insightsonindia.com 63 Insights IAS

• The wind blows from high pressure to low pressure. The wind at the surface

experiences friction. In addition, rotation of the earth also affects the wind

movement. The force exerted by the rotation of the earth is known as the

Coriolis force. Thus, the horizontal winds near the earth surface respond to

the combined effect of three forces – the pressure gradient force, the frictional

force and the Coriolis force. In addition, the gravitational force acts downward.

Coriolis Force

• The rotation of the earth about its axis affects the direction of the wind. This

force is called the Coriolis force after the French physicist who described it in

1844. It deflects the wind to the right direction in the northern hemisphere and

to the left in the southern hemisphere. The deflection is more when the wind

velocity is high. The Coriolis force is directly proportional to the angle of

latitude. It is maximum at the poles and is absent at the equator.

• The Coriolis force acts perpendicular to the pressure gradient force. The

pressure gradient force is perpendicular to an isobar. The higher the pressure

gradient force, the more is the velocity of the wind and the larger is the

deflection in the direction of wind. As a result of these two forces operating

perpendicular to each other, in the low-pressure areas the wind blows around

it. At the equator, the Coriolis force is zero and the wind blows perpendicular to

the isobars. The low pressure gets filled instead of getting intensified. That is

the reason why tropical cyclones are not formed near the equator.

65. Which of the following vegetations can be found in Nilgiri Biosphere Reserve

1. Swamps

2. Dry and moist deciduous

3. Wet evergreen forests

4. Dry scrubs

Select the correct answer using the code given below:

(a) 1, 2 and 3 only

(b) 2, 3 and 4 only

(c) 1, 3 and 4 only

(d) 1, 2, 3 and 4

Solution: D

The Nilgiri Biosphere Reserve possesses different habitat types, unspoilt areas of

natural vegetation types with several dry scrubs, dry and moist deciduous, semi-

evergreen and wet evergreen forests, evergreen sholas, grasslands and swamps. It

includes the largest known population of two endangered animal species, namely the

Nilgiri Tahr and the Lion-tailed macaque. The largest south Indian population of

elephant, tiger, gaur, sambar and chital as well as a good number of endemic and

Page 66: SIMPLYFYING IAS EXAM PREPARATION - INSIGHTSIAS · 2020. 4. 7. · Insta 75 Days Revision Plan for UPSC Civil Services Prelims – 2020 This document is the compilation of 100 questions

INSTA 75 Days REVISION PLAN for Prelims 2020 - InstaTests

www.insightsonindia.com 64 Insights IAS

endangered plants are also found in this reserve. The habitat of a number of tribal

groups remarkable for their traditional modes of harmonious use of the environment

are also found here.

Extra Learning:

• Biosphere reserves are sites established by countries and recognized under

UNESCO’s Man and the Biosphere (MAB) Programme to promote

sustainable development based on local community efforts and sound science.

The programme of Biosphere Reserve was initiated by UNESCO in 1971. The

purpose of the formation of the biosphere reserve is to conserve in situ all forms

of life, along with its support system, in its totality, so that it could serve as a

referral system for monitoring and evaluating changes in natural ecosystems.

The first biosphere reserve of the world was established in 1979, since then the

network of biosphere reserves has increased to 631 in 119 countries across the

world

66. The Community Policing Scheme ‘Prahari’ has been launched by

(a) Kerala

(b) Gujarat

(c) Haryana

(d) Delhi

Solution: D

Community policing scheme ‘Prahari’:

• Launched by Delhi Police.

• To prevent crime and maintain law and order in Delhi.

Page 67: SIMPLYFYING IAS EXAM PREPARATION - INSIGHTSIAS · 2020. 4. 7. · Insta 75 Days Revision Plan for UPSC Civil Services Prelims – 2020 This document is the compilation of 100 questions

INSTA 75 Days REVISION PLAN for Prelims 2020 - InstaTests

www.insightsonindia.com 65 Insights IAS

• Under this scheme, civil personnel such as chowkidars and security guards

deployed in the commercial and VIP areas will act as the eyes and ears of the

police.

• Significance: The move will help to bring professionalism in the work of security

guards by giving them appropriate training and enabling frequent police-Prahari

interaction and co-operation.

https://www.insightsonindia.com/2019/10/19/insights-daily-current-affairs-pib-19-

october-2019/

67. Consider the following pairs regarding the local winds with the places of their

origin

1. Fohn: Andes

2. Chinook: Rockies

3. Harmattan: Guinea Coast

4. Zonda: Alps

Which of the pairs given above is/are correctly matched?

(a) 1 and 2 only

(b) 2 and 3 only

(c) 3 and 4 only

(d) 1 and 4 only

Solution: B

Differences in the heating and cooling of earth surfaces and the cycles those develop

daily or annually can create several common, local or regional winds.

Local winds

Page 68: SIMPLYFYING IAS EXAM PREPARATION - INSIGHTSIAS · 2020. 4. 7. · Insta 75 Days Revision Plan for UPSC Civil Services Prelims – 2020 This document is the compilation of 100 questions

INSTA 75 Days REVISION PLAN for Prelims 2020 - InstaTests

www.insightsonindia.com 66 Insights IAS

68. Which of the following tree species belong to the Moist deciduous forests

1. Teak

2. Sal

3. Amla

4. Sandalwood

Select the correct answer using the code given below:

(a) 1, 2 and 3 only

(b) 2, 3 and 4 only

(c) 1, 3 and 4 only

(d) 1, 2, 3 and 4

Solution: D

• Tropical Deciduous forests are the most widespread forests in India.

• They are also called the monsoon forests. They spread over regions which

receive rainfall between 70-200 cm. On the basis of the availability of water,

these forests are further divided into moist and dry deciduous.

• The Moist deciduous forests are more pronounced in the regions which

record rainfall between 100-200 cm. These forests are found in the

northeastern states along the foothills of Himalayas, eastern slopes of the

Western Ghats and Odisha. Teak, sal, shisham, hurra, mahua, amla, semul,

kusum, and sandalwood etc. are the main species of these forests.

Extra Learning:

On the basis of certain common features such as predominant vegetation type and

climatic regions, Indian forests can be divided into the following groups:

TYPES OF FORESTS:

(i) Tropical Evergreen and Semi Evergreen forests

(ii) Tropical Deciduous forests

(iii) Tropical Thorn forests

(iv) Montane forests

(v) Littoral and Swamp forests.

Page 69: SIMPLYFYING IAS EXAM PREPARATION - INSIGHTSIAS · 2020. 4. 7. · Insta 75 Days Revision Plan for UPSC Civil Services Prelims – 2020 This document is the compilation of 100 questions

INSTA 75 Days REVISION PLAN for Prelims 2020 - InstaTests

www.insightsonindia.com 67 Insights IAS

69. Consider the following statements regarding Mount Kilimanjaro

1. It is a dormant volcano in Nairobi.

2. It is the highest mountain in Africa.

Which of the statements given above is/are correct?

(a) 1 only

(b) 2 only

(c) Both 1 and 2

(d) Neither 1 nor 2

Page 70: SIMPLYFYING IAS EXAM PREPARATION - INSIGHTSIAS · 2020. 4. 7. · Insta 75 Days Revision Plan for UPSC Civil Services Prelims – 2020 This document is the compilation of 100 questions

INSTA 75 Days REVISION PLAN for Prelims 2020 - InstaTests

www.insightsonindia.com 68 Insights IAS

Solution: B

Mount Kilimanjaro:

• Differently-abled man from Ernakulam district scales Mount Kilimanjaro.

• Mount Kilimanjaro is a dormant volcano in Tanzania.

• It is the highest mountain in Africa, with its summit about 4,900 metres (16,100

ft) from its base, and 5,895 metres (19,341 ft) above sea level.

• The first people known to have reached the summit of the mountain were Hans

Meyer and Ludwig Purtscheller, in 1889.

https://www.insightsonindia.com/2019/10/18/insights-daily-current-affairs-pib-18-

october-2019/

70. Consider the following statements regarding Nor Westers

1. These are pre-monsoon showers which are a common phenomenon in

Kerala and coastal areas of Karnataka.

2. These showers are useful for tea, jute and rice cultivation.

Which of the statements given above is/are correct?

(a) 1 only

(b) 2 only

(c) Both 1 and 2

Page 71: SIMPLYFYING IAS EXAM PREPARATION - INSIGHTSIAS · 2020. 4. 7. · Insta 75 Days Revision Plan for UPSC Civil Services Prelims – 2020 This document is the compilation of 100 questions

INSTA 75 Days REVISION PLAN for Prelims 2020 - InstaTests

www.insightsonindia.com 69 Insights IAS

(d) Neither 1 nor 2

Solution: B

Nor Westers: These are dreaded evening thunderstorms in Bengal and Assam.

Their notorious nature can be understood from the local nomenclature of

‘Kalbaisakhi’, a calamity of the month of Baisakh. These showers are useful for tea,

jute and rice cultivation. In Assam, these storms are known as “Bardoli Chheerha”.

Extra Learning:

Some Famous Local Storms of Hot Weather Season:

(i) Mango Shower : Towards the end of summer, there are pre-monsoon

showers which are a common phenomenon in Kerala and coastal areas of

Karnataka. Locally, they are known as mango showers since they help in the

early ripening of mangoes.

(ii) Blossom Shower : With this shower, coffee flowers blossom in Kerala and

nearby areas.

(iii) Loo : Hot, dry and oppressing winds blowing in the Northern plains from

Punjab to Bihar with higher intensity between Delhi and Patna.

71. Arrange the following cities of West Asia from East to west

1. Bandar Abbas

2. Mosul

3. Aleppo

4. Manama

Select the correct answer using the code given below?

(a) 3 2 4 1

(b) 4 2 3 1

(c) 1 4 2 3

(d) 1 3 2 4

Solution: C

Extra Learning:

Bandar Abbas or Bandar-e ‘Abbās is a port city on the southern coast of Iran, on the

Persian Gulf. The city occupies a strategic position on the narrow Strait of Hormuz,

and it is the location of the main base of the Iranian Navy.

Page 72: SIMPLYFYING IAS EXAM PREPARATION - INSIGHTSIAS · 2020. 4. 7. · Insta 75 Days Revision Plan for UPSC Civil Services Prelims – 2020 This document is the compilation of 100 questions

INSTA 75 Days REVISION PLAN for Prelims 2020 - InstaTests

www.insightsonindia.com 70 Insights IAS

Aleppo is a city in Syria, which serves as the capital of the Aleppo Governorate, the

most populous Syrian governorate. With an official population of 4.6 million in 2010,

Aleppo was the largest Syrian city before the Syrian Civil War; however, it is now the

second-largest city in Syria, after the capital Damascus.

Mosul is a major city in northern Iraq. Located approximately 400 km north of

Baghdad, Mosul stands on the west bank of the Tigris, opposite the ancient Assyrian

city of Nineveh on the east bank. Mosul fell to IS in June 2014. The battle for Mosul

was a huge military operation to recapture Iraq’s second largest city from so-called

Islamic State (IS).

Manama, the modern capital of the Arabian Gulf island nation of Bahrain, has been

at the center of major trade routes since antiquity.

72. Who among the following ministry/organizations has launched RUCO

(Repurpose Used Cooking Oil) initiative?

(a) NITI Aayog

(b) Ministry of Petroleum and Natural Gas

Page 73: SIMPLYFYING IAS EXAM PREPARATION - INSIGHTSIAS · 2020. 4. 7. · Insta 75 Days Revision Plan for UPSC Civil Services Prelims – 2020 This document is the compilation of 100 questions

INSTA 75 Days REVISION PLAN for Prelims 2020 - InstaTests

www.insightsonindia.com 71 Insights IAS

(c) Food Safety and Standards Authority of India (FSSAI)

(d) Council of Scientific & Industrial Research (CSIR),

Solution: C

To mark the 150th birth anniversary of Mahatma Gandhi on 2nd October as Rashtriya

Swachhta Diwas, publicity vans were flagged off in Delhi, to generate awareness

among the people about the OMCs’ initiative of converting Used Cooking Oil to

Biodiesel. The publicity involves wide social media campaign to spread awareness

and educate people about the ill effects of used cooking oil and ways to dispose it off

for converting it to biodiesel.

Concerns associated with the consumption of used cooking oil (UCO):

• During frying, several properties of oil are altered, Total Polar

Compounds(TPC) are formed on repeated frying.

• The toxicity of these compounds is associated with several diseases such as

hypertension, atherosclerosis, Alzheimer’s disease, liver diseases.

Limit set by FSSAI:

FSSAI has fixed a limit for Total Polar Compounds at 25 percent beyond which the

vegetable oil shall not be used.

What are Total Polar Compounds (TPC)?

The level of TPC increases every time oil is re-heated.

TPC accumulation in oil without food is slower than that in oil frying with food.

About RUCO:

Launched by FSSAI.

• The initiative will enable collection and conversion of used cooking oil to bio-

diesel.

• FSSAI wants businesses using more than 100 litres of oil for frying, to maintain

a stock register and ensure that RUCO is handed over to only registered

collecting agencies.

Potential and significance of the initiative:

• India has the potential to recover 220 crore litres of used cooking oil for the

production of biodiesel by 2022 through a co-ordinated action.

• While biodiesel produced from used cooking oil is currently very small, but a

robust ecosystem for conversion and collection is rapidly growing in India and

will soon reach a sizable scale.

Page 74: SIMPLYFYING IAS EXAM PREPARATION - INSIGHTSIAS · 2020. 4. 7. · Insta 75 Days Revision Plan for UPSC Civil Services Prelims – 2020 This document is the compilation of 100 questions

INSTA 75 Days REVISION PLAN for Prelims 2020 - InstaTests

www.insightsonindia.com 72 Insights IAS

https://www.insightsonindia.com/2019/10/02/ruco-repurpose-used-cooking-oil-

initiative-2/

73. Consider the following statements regarding National Disaster Response Fund

(NDRF)

1. Comptroller and Auditor General of India (CAG) audit the accounts of

NDRF.

2. It constituted under the Disaster Management Act, 2005.

3. It forms the part Public Accounts of Government of India

Which of the statements given above is/are correct?

(a) 2 and 3 only

(b) 1 and 3 only

(c) 1 and 2 only

(d) 1, 2 and 3

Solution: D

Centre approves additional financial assistance of Rs.1813.75 crore from National

Disaster Response Fund (NDRF) to the State of Karnataka and Bihar.

Background:

• The States of Karnataka and Bihar have apprised of the paucity of funds in the

SDRF account, resulting in delay in providing relief assistance to the affected

people and requested for release of advance additional financial assistance

from NDRF.

About NDRF:

• Defined in Section 46 of the Disaster Management Act, 2005 (DM Act).

• It is a fund managed by the Central Government for meeting the expenses for

emergency response, relief and rehabilitation due to any threatening disaster

situation or disaster.

• Constituted to supplement the funds of the State Disaster Response Funds

(SDRF) of the states to facilitate immediate relief in case of calamities of a

severe nature.

• National Calamity Contingency Fund (NCCF) was renamed as National

Disaster Response Fund (NDRF) with the enactment of the Disaster

Management Act in 2005.

What is it to be used for?

• NDRF amount can be spent only towards meeting the expenses for emergency

response, relief and rehabilitation.

Page 75: SIMPLYFYING IAS EXAM PREPARATION - INSIGHTSIAS · 2020. 4. 7. · Insta 75 Days Revision Plan for UPSC Civil Services Prelims – 2020 This document is the compilation of 100 questions

INSTA 75 Days REVISION PLAN for Prelims 2020 - InstaTests

www.insightsonindia.com 73 Insights IAS

• For projects exclusively for the purpose of mitigation, i.e, measures aimed at

reducing the risk, impact or effect of a disaster or threatening disaster situation

a separate fund called National Disaster Mitigation Fund has to be constituted.

Sources of Financing NDRF:

• Financed through the levy of a cess on certain items, chargeable to excise and

customs duty, and approved annually through the Finance Bill.

• The requirement for funds beyond what is available under the NDRF is met

through general budgetary resources.

• Currently, a National Calamity Contingency Duty (NCCD) is levied to finance

the NDRF and additional budgetary support is provided as and when

necessary.

• A provision also exists in the DM Act to encourage any person or institution to

make a contribution to the NDRF.

Key features of NDRF:

• Located in the “Public Accounts” of Government of India under “Reserve

Funds not bearing interest“.

• Monitoring: Department of Agriculture and Cooperation under Ministry of

Agriculture (MoA) monitors relief activities for calamities associated with

drought, hailstorms, pest attacks and cold wave /frost while rest of the natural

calamities are monitored by Ministry of Home Affairs (MHA).

• Comptroller and Auditor General of India (CAG) audits the accounts of

NDRF.

https://www.insightsonindia.com/2019/10/05/national-disaster-response-fund-ndrf/

74. Consider the following statements regarding Fronts

1. When an air mass is fully lifted above the land surface, it is called the

occluded front.

2. When the cold air moves towards the warm air mass, its contact zone is

called the warm front.

3. When the warm air mass moves towards the cold air mass, the contact

zone is a cold front.

Which of the statements given above is/are correct?

(a) 1 only

(b) 2 and 3 only

(c) 1 and 3 only

(d) 1, 2 and 3

Solution: A

Page 76: SIMPLYFYING IAS EXAM PREPARATION - INSIGHTSIAS · 2020. 4. 7. · Insta 75 Days Revision Plan for UPSC Civil Services Prelims – 2020 This document is the compilation of 100 questions

INSTA 75 Days REVISION PLAN for Prelims 2020 - InstaTests

www.insightsonindia.com 74 Insights IAS

The air with distinctive characteristics in terms of temperature and humidity is called

an airmass. It is defined as a large body of air having little horizontal variation in

temperature and moisture. The homogenous surfaces, over which air masses form,

are called the source regions.

When the cold air moves towards the warm air mass, its contact zone is called the

cold front, whereas if the warm air mass moves towards the cold air mass, the contact

zone is a warm front. If an air mass is fully lifted above the land surface, it is called

the occluded front. The fronts occur in middle latitudes and are characterized by

steep gradient in temperature and pressure. They bring abrupt changes in

temperature and cause the air to rise to form clouds and cause precipitation.

75. Consider the following statements regarding Guru Ravidas

1. His devotional songs were included in the Guru Granth Sahib.

2. He taught removal of social divisions of caste and gender, and promoted

unity in the pursuit of personal spiritual freedom.

Which of the statements given above is/are correct?

(a) 1 only

(b) 2 only

Page 77: SIMPLYFYING IAS EXAM PREPARATION - INSIGHTSIAS · 2020. 4. 7. · Insta 75 Days Revision Plan for UPSC Civil Services Prelims – 2020 This document is the compilation of 100 questions

INSTA 75 Days REVISION PLAN for Prelims 2020 - InstaTests

www.insightsonindia.com 75 Insights IAS

(c) Both 1 and 2

(d) Neither 1 nor 2

Solution: C

Yielding to the protests against the Supreme Court-directed demolition of a Guru

Ravidas temple in south Delhi, the Centre has agreed to allot the same site to the

devotees to construct a new temple.

Who is Guru Ravidas?

• He was a poet and saint of the Bhakti movement.

• Born in Varanasi to the ‘untouchable’ leather tanners caste.

• He is believed to be a disciple of the bhakti saint-poet Ramanandaand a

contemporary of the bhakti saint-poet Kabir.

• One of his famous disciples was the saint, Mirabai.

• Ravidas’ devotional songs were included in the Sikh scriptures, Guru

Granth Sahib.

• The Panch Vani text of the Dadupanthi tradition within Hinduism also

includes numerous poems of Ravidas.

• Ravidas taught removal of social divisions of caste and gender, and

promoted unity in the pursuit of personal spiritual freedoms.

• Among Ravidas’s moral and intellectual achievements were the conception of

“Begampura”, a city that knows no sorrow; and a society where caste and class

have ceased to matter.

https://www.insightsonindia.com/2019/10/19/guru-ravidas-2/

DAY – 16 (InstaTest-16)

76. Consider the following statements regarding Corruption Perceptions Index

1. It ranks around 180 countries and territories by their perceived levels of

public sector corruption.

2. It is an index published annually by Transparency International

Which of the statements given above is/are correct?

(a) 1 only

(b) 2 only

(c) Both 1 and 2

(d) Neither 1 nor 2

Page 78: SIMPLYFYING IAS EXAM PREPARATION - INSIGHTSIAS · 2020. 4. 7. · Insta 75 Days Revision Plan for UPSC Civil Services Prelims – 2020 This document is the compilation of 100 questions

INSTA 75 Days REVISION PLAN for Prelims 2020 - InstaTests

www.insightsonindia.com 76 Insights IAS

Solution: C

The Corruption Perceptions Index is an index published annually by Transparency

International since 1995 which ranks countries “by their perceived levels of public

sector corruption, as determined by expert assessments and opinion surveys.

India has been ranked at the 80th position among 180 countries and territories in

the Corruption Perception Index (CPI) prepared by Transparency International.

Denmark and New Zealand have cornered the top spot, followed by Finland,

Singapore, Sweden and Switzerland in the top ten.

77. Consider the following statements regarding Inner Line Permit (ILP)

1. It is a document required by non- natives to visit or stay in a state that is

protected under the ILP system.

2. At present, it is required only in Arunachal Pradesh and Nagaland.

3. The ILP is issued by the central government.

Which of the statements given above is/are correct?

(a) 1 only

(b) 2 and 3 only

(c) 1 and 3 only

(d) 1, 2 and 3

Solution: A

Inner Line Permit (ILP)

• It is a document required by non- natives to visit or stay in a state that is

protected under the ILP system.

• At present, four Northeastern states are covered, namely, Arunachal Pradesh,

Mizoram, Manipur and Nagaland. (The inclusion of Manipur in Inner Line

Permit was announced on 10 December 2019)

• Both the duration of stay and the areas allowed to be accessed for any non-

native are determined by the ILP.

• The ILP is issued by the concerned state government and can be availed both

by applying online or in person.

Extra Learning:

HISTORY of ILP:

• The Inner Line Permit is an extension of the Bengal Eastern Frontier

Regulation Act 1873. The Britishers framed regulations restricting entry in

Page 79: SIMPLYFYING IAS EXAM PREPARATION - INSIGHTSIAS · 2020. 4. 7. · Insta 75 Days Revision Plan for UPSC Civil Services Prelims – 2020 This document is the compilation of 100 questions

INSTA 75 Days REVISION PLAN for Prelims 2020 - InstaTests

www.insightsonindia.com 77 Insights IAS

certain designated areas. This was done to protect the Crown’s interest in

certain states by preventing “British subjects” (Indians) from trading within these

regions.

• In 1950, the term ‘British subjects’ was replaced with ‘Citizens of India’. Today,

all non-natives require the permit. This was done to protect the indigenous tribal

communities of these states from exploitation.

What about foreigners?

An ILP is only valid for domestic tourists. For foreign tourists in:

• Manipur: No permit is required. But, have to register themselves.

• Mizoram: No permit is required. But, need to register.

• Nagaland: No permit is required. However, they need to register.

• Arunachal Pradesh: Tourists need a Protected Area Permit (PAP) or Restricted

Area Permit (RAP) from the Ministry of Home Affairs, Government of India.

https://www.insightsonindia.com/2020/03/02/inner-line-permit-ilp-2/

78. Consider the following statements regarding National Anthem of India

1. The National Anthem was first sung at the 1911 Calcutta session of the

Congress.

2. ‘Jana Gana Mana’ was adopted as the country’s National Anthem by the

Constituent Assembly of India on 16th August 1947 soon after

Independence.

Which of the statements given above is/are correct?

(a) 1 only

(b) 2 only

(c) Both 1 and 2

(d) Neither 1 nor 2

Solution: A

About National Anthem :

• On December 27, 1911, the National Anthem was first sung at the Calcutta

session of the Congress. In 1912, the song was published under the title Bharat

Bidhata in the Tatwabodhini Patrika, which was the official publication of the

Brahmo Samaj and of which Tagore was the Editor.

• ‘Jana Gana Mana’ was adopted as the country’s National Anthem by the

Constituent Assembly of India on January 24, 1950, the last day of its last

session.

Page 80: SIMPLYFYING IAS EXAM PREPARATION - INSIGHTSIAS · 2020. 4. 7. · Insta 75 Days Revision Plan for UPSC Civil Services Prelims – 2020 This document is the compilation of 100 questions

INSTA 75 Days REVISION PLAN for Prelims 2020 - InstaTests

www.insightsonindia.com 78 Insights IAS

Extra Learning:

• Reverence to the National Anthem is a Fundamental duty as per Article

51A (a) of the Constitution.

• Originally written in Bengali, it is the first of five stanzas of a poem written and

later set to notations by Rabindranath Tagore.

• The members of the Indian Delegation to the General Assembly of the United

Nations held at New York in 1947 gave a recording of Jana Gana Mana as the

country’s national anthem.

1) The full version of the Anthem shall be played on the following occasions: –

i) Civil and Military investitures;

ii) When National Salute (which means the Command “Rashtriya Salute –

Salami Shastr” to the accompaniment of the National Anthem is given on

ceremonial occasions to the President or to the Governor/Lieutenant

Governor within their respective States/ Union Territories;

iii) During parades – irrespective of whether any of the dignitaries referred to in

(ii) above is present or not;

iv) On arrival of the President at formal State functions and other functions

organized by the Government and mass functions and on his departure from

such functions;

v) Immediately before and after the President addresses the Nation over All

India Radio;

vi) On arrival of the Governor/Lieutenant Governor at formal State functions

within his State/Union Territory and on his departure from such functions;

vii) When the National Flag is brought on parade;

viii) When the Regimental Colours are presented;

ix) For hoisting of colours in the Navy.

2) The short version of the Anthem shall be played when drinking toasts in Messes.

3) The Anthem shall be played on any other occasion for which special orders have

been issued by the Government of India.

4) Normally the Anthem shall not be played for the Prime Minister, though there may

be special occasions when it may be played.

5) When the National Anthem is played by a band, the Anthem will be preceded by

a roll of drums to assist the audience to know that the National Anthem is going to

be played

A specific set of rules and regulations have been set by the Government of India to

oversee the proper and correct rendition of the National Anthem. The Prevention of

Insults to National Honour Act, 1971, was penned down by the Government of India

to prevent any intentional disrespect or insult towards the National Anthem of the

country. Offenders are punishable with up to three years of imprisonment as well as

monetary fine.

https://mha.gov.in/sites/default/files/NationalAnthem%28E%29_2.pdf

Page 81: SIMPLYFYING IAS EXAM PREPARATION - INSIGHTSIAS · 2020. 4. 7. · Insta 75 Days Revision Plan for UPSC Civil Services Prelims – 2020 This document is the compilation of 100 questions

INSTA 75 Days REVISION PLAN for Prelims 2020 - InstaTests

www.insightsonindia.com 79 Insights IAS

79. Which of the following is/are the applications of Artificial Intelligence?

1. Assistance in surgeries

2. Personalized shopping

3. Financial Accounting

Select the correct answer using the code given below

(a) 1 and 3 only

(b) 2 and 3 only

(c) 2 only

(d) 1, 2 and 3

Solution: D

• Artificial intelligence (AI) is the ability of a computer program or a machine to

think and learn. It is also a field of study which tries to make computers “smart”.

• They work on their own without being encoded with commands.John McCarthy

came up with the name “artificial intelligence” in 1955.

• The term may also be applied to any machine that exhibits traits associated

with a human mind such as learning and problem-solving.

It has lot of applications in different fields like

• Financial Reporting

• Personalized shopping

• Assistance in surgeries

• Deep Learning

• Cyber Security

• Intelligence Gathering etc.

https://becominghuman.ai/10-powerful-examples-of-ai-applications-553f7f062d9f

Extra Reading:

Page 82: SIMPLYFYING IAS EXAM PREPARATION - INSIGHTSIAS · 2020. 4. 7. · Insta 75 Days Revision Plan for UPSC Civil Services Prelims – 2020 This document is the compilation of 100 questions

INSTA 75 Days REVISION PLAN for Prelims 2020 - InstaTests

www.insightsonindia.com 80 Insights IAS

80. Consider the following statements regarding Central Waqf Council (CWC)

1. It is a non-statutory body set up by executive resolution for the purpose

of advising Centre on matters pertaining to working of the State Wakf

Boards and proper administration of the Wakfs in the country.

2. The secretary to Ministry of Minority affairs is the ex-officio chairperson

of the council.

Which of the statements given above is/are correct?

(a) 1 only

(b) 2 only

(c) Both 1 and 2

(d) Neither 1 nor 2

Solution: D

Central Waqf Council (CWC)

• Central Waqf Council is a statutory body under the administrative control of

the Ministry of Minority Affairs was set up in 1964 as per the provision given

in the Waqf Act, 1954 as Advisory Body to the Central Government on matters

concerning the working of the Waqf Boards and the due administration of

Auqaf.

• However, the role of the Council was expanded significantly under the

provisions of the Waqf (Amendment) Act, 2013. The Council has been

empowered to advise the Central Government, State Governments and State

Waqf Boards.

• It will now issue directives to the boards/ State Government to furnish

information to the Council on the performance of the board particularly on their

financial performance, survey, revenue records, encroachment of Waqf

properties, Annual and Audit report etc under section 9 (4) of the act.

• It is a permanent dedication of movable or immovable properties for religious,

pious or charitable purposes as recognized by Muslim Law, given by

philanthropists.

• The Council is headed by a Chairperson, who is the Union Minister in charge

of Wakfs and there are maximum 20 other members, appointed by Government

of India as stipulated in the Wakf Act.

Extra Learning:

Major Functions of Central Waqf Council

• To advise Central Government, State Governments, State Waqf Boards on

matters concerning the working of the Boards and due administration of Auqaf.

Page 83: SIMPLYFYING IAS EXAM PREPARATION - INSIGHTSIAS · 2020. 4. 7. · Insta 75 Days Revision Plan for UPSC Civil Services Prelims – 2020 This document is the compilation of 100 questions

INSTA 75 Days REVISION PLAN for Prelims 2020 - InstaTests

www.insightsonindia.com 81 Insights IAS

• To monitor the implementation of the provisions of Waqf (Amendment) Act,

2013 in States and UTs.

• To render legal advice on protection and retrieval of the Waqf Properties and

for removal of encroachment etc.

• To implement the Scheme for Development of Urban Waqf Properties &

Identification of potential Waqf land for development by National Waqf

Development Corporation Ltd.

• To implement Educational and Women Welfare Schemes for skill development

and to empower the poor, especially Women.

• To implement the Scheme of Computerization of the State Waqf Boards

records, a Central sector scheme of Ministry of Minority Affairs.

• To seek necessary information from the State Government/Boards on the

performance of the State Waqf Boards as per the provision given in the Waqf

(Amendment) Act, 2013.

• To take up the Waqf matters with various departments of Central and State

Governments such as ASI, Railways, Revenue and Forest etc.

• To undertake awareness programmes to promote the interest of the Council

and to sensitize the Waqf institutions about their new roles and responsibilities.

81. Consider the following statements:

1. Reservation in promotion in public posts is a fundamental right.

2. Article 335 recognizes that special measures need to be adopted for

considering the claims of SCs and STs in order to bring them to a level-

playing field.

Which of the statements given above is/are correct?

(a) 1 only

(b) 2 only

(c) Both 1 and 2

(d) Neither 1 nor 2

Solution: B

Reservation in promotion in public posts not a fundamental right: SC.

• The Supreme Court has recently ruled that the states are not bound to provide

reservation in appointments and promotions and that there is no fundamental

right to reservation in promotions.

What has the court said?

• Reservation in promotion in public posts cannot be claimed as a fundamental

right.

Page 84: SIMPLYFYING IAS EXAM PREPARATION - INSIGHTSIAS · 2020. 4. 7. · Insta 75 Days Revision Plan for UPSC Civil Services Prelims – 2020 This document is the compilation of 100 questions

INSTA 75 Days REVISION PLAN for Prelims 2020 - InstaTests

www.insightsonindia.com 82 Insights IAS

• Articles 16 (4) and 16

(4-A) of the

Constitution does not

confer individuals with

a fundamental right to

claim reservation in

promotion. It only

empowers the State to

make a reservation in

matters of appointment

and promotion in favour

of the Scheduled

Castes and the

Scheduled Tribes, only if in the opinion of the State they are not adequately

represented in the services of the State.

• State governments are not bound to make a reservation and have

discretion in providing reservations.

• The judgment also noted that even the courts could not issue a mandamus

directing the States to provide reservation.

• Article 335 recognizes that special measures need to be adopted for

considering the claims of SCs and STs in order to bring them to a level-playing

field.

• In its landmark 1992 decision in Indra Sawhney vs Union of India, the

Supreme Court had held that reservations under Article 16(4) could only be

provided at the time of entry into government service but not in matters

of promotion.

• Supreme Court added that the principle would operate only prospectively

and not affect promotions already made and that reservation already provided

in promotions shall continue in operation for a period of five years from the date

of the judgment. It also ruled that the creamy layer can be and must be

excluded.

• On June 17, 1995, Parliament, acting in its constituent capacity, adopted the

seventy-seventh amendment by which clause (4A) was inserted into Article

16 to enable reservation to be made in promotion for SCs and STs. The validity

of the seventy-seventh and eighty-fifth amendments to the Constitution and of

the legislation enacted in pursuance of those amendments was challenged

before the Supreme Court in the Nagaraj case.

• Upholding the validity of Article 16 (4A), the court then said that it is an

enabling provision. “The State is not bound to make reservation for the SCs

and STs in promotions. But, if it seeks to do so, it must collect quantifiable data

on three facets — the backwardness of the class; the inadequacy of the

representation of that class in public employment; and the general efficiency

of service as mandated by Article 335 would not be affected”.

• The court ruled that the constitutional amendments do not abrogate the

fundamentals of equality.

Page 85: SIMPLYFYING IAS EXAM PREPARATION - INSIGHTSIAS · 2020. 4. 7. · Insta 75 Days Revision Plan for UPSC Civil Services Prelims – 2020 This document is the compilation of 100 questions

INSTA 75 Days REVISION PLAN for Prelims 2020 - InstaTests

www.insightsonindia.com 83 Insights IAS

https://www.insightsonindia.com/2020/02/10/reservation-in-promotion-in-public-

posts-not-a-fundamental-right-sc/

82. Which is the first state to pass a law against lynching in India?

(a) Kerala

(b) Manipur

(c) West Bengal

(d) Assam

Solution: B

• Manipur became the first to pass a remarkable law against lynching. The

Manipur law closely follows the Supreme Court’s prescriptions, creating a nodal

officer to control such crimes in every State, special courts and enhanced

punishments.

What is Lynching?

• Lynching is a premeditated extrajudicial killing by a group. It is most often used

to characterize informal public executions by a mob in order to punish an

alleged transgressor, punish a convicted transgressor, or intimidate a group.

https://www.thehindu.com/opinion/lead/manipur-shows-the-way/article26007016.ece

83. Consider the following statements regarding National Security Act

1. It is a law that allows punitive detention, if authorities are satisfied that a

person is a threat to national security or law and order.

2. The person does not need to be charged during this period of detention.

Which of the statements given above is/are correct?

(a) 1 only

(b) 2 only

(c) Both 1 and 2

(d) Neither 1 nor 2

Solution: B

National Security Act

• It is a stringent law that allows preventive detention for months, if authorities

are satisfied that a person is a threat to national security or law and order.

Page 86: SIMPLYFYING IAS EXAM PREPARATION - INSIGHTSIAS · 2020. 4. 7. · Insta 75 Days Revision Plan for UPSC Civil Services Prelims – 2020 This document is the compilation of 100 questions

INSTA 75 Days REVISION PLAN for Prelims 2020 - InstaTests

www.insightsonindia.com 84 Insights IAS

• The person does not need to be charged during this period of detention.

• The goal is to prevent the individual from committing a crime.

• It was promulgated on September 23, 1980, during the Indira Gandhi

government.

As per the National Security Act, the grounds for preventive detention of a person

include:

• acting in any manner prejudicial to the defense of India, the relations of India

with foreign powers, or the security of India.

• regulating the continued presence of any foreigner in India or with a view to

making arrangements for his expulsion from India.

• preventing them from acting in any manner prejudicial to the security of the

State or from acting in any manner prejudicial to the maintenance of public

order or from acting in any manner prejudicial to the maintenance of supplies

and services essential to the community it is necessary so to do.

Duration:

• Under the National Security Act, an individual can be detained without a

charge for up to 12 months; the state government needs to be intimated that

a person has been detained under the NSA.

• A person detained under the National Security Act can be held for 10 days

without being told the charges against them.

Appeal: The detained person can appeal before a high court advisory board but they

are not allowed a lawyer during the trial.

Extra Learning:

Public Safety Act of Jammu and Kashmir:

• The Jammu and Kashmir Public Safety Act, 1978 (PSA) is a preventive

detention law under which a person is taken into custody to prevent them from

acting harmfully against “the security of the state or the maintenance of the

public order” in the Indian state of Jammu and Kashmir (now a union territory).

Whereas PSA applies only to Jammu and Kashmir, it is very similar to the

National Security Act that is used by the central and other state governments

of India for preventive detention

• Within four weeks of passing the detention order, the government has to refer

the case to an Advisory Board. This Advisory Board will have to give its

recommendations within eight weeks of the order. If the Board thinks that there

is cause for preventive detention, the government can hold the person up to

two years.

• Usually when a person is arrested, they have the right to legal representation

and can challenge the arrest. But, when a person is arrested under the PSA,

they do not have these rights before the Advisory Board unless sufficient

Page 87: SIMPLYFYING IAS EXAM PREPARATION - INSIGHTSIAS · 2020. 4. 7. · Insta 75 Days Revision Plan for UPSC Civil Services Prelims – 2020 This document is the compilation of 100 questions

INSTA 75 Days REVISION PLAN for Prelims 2020 - InstaTests

www.insightsonindia.com 85 Insights IAS

grounds can be established that the detention is illegal. There have been cases

where the High Court has interfered and quashed the detention.

• According to Section 13(2), the detaining authority need not even inform the

detained individual as to the reason for the action, if it decides that it goes

against public interest.

• The Jammu and Kashmir Public Safety Act (PSA) received the assent of the

J&K Governor on April 8, 1978.

• The Act was introduced as a tough law to prevent the smuggling of timber and

keep the smugglers “out of circulation”.

• The law allows the government to detain any person above the age of 16

without trial for a period of two years.

• The PSA allows for administrative detention for up to two years “in the case of

persons acting in any manner prejudicial to the security of the State”, and for

administrative detention up to one year where “any person is acting in any

manner prejudicial to the maintenance of public order”.

• Detention orders under PSA can be issued by Divisional Commissioners or

District Magistrates.

• Section 22 of the Act provides protection for any action taken “in good faith”

under the Act: “No suit, prosecution or any other legal proceeding shall lie

against any person for anything done or intended to be done in good faith in

pursuance of the provisions of this Act.”

• Under Section 23 of the Act, the government is empowered to “make such

Rules consistent with the provisions of this Act, as may be necessary for

carrying out the objects of this Act”.

• In August 2018, the Act was amended to allow individuals to be detained under

the PSA outside the state as well.

https://www.insightsonindia.com/2020/02/07/public-safety-act/

84. Consider the following statements regarding Section 144 of the Criminal

Procedure Code (Cr.PC)

1. It is imposed by District Magistrate, a sub- divisional Magistrate or any

other Executive Magistrate on behalf of the State Government.

2. The order under Section 144 shall remain in force indefinitely until

withdrawn by the government.

3. Section 144 empowers the authorities to block the internet access.

Which of the statements given above is/are correct?

(a) 1 only

(b) 2 and 3 only

(c) 1 and 3 only

(d) 1, 2 and 3

Page 88: SIMPLYFYING IAS EXAM PREPARATION - INSIGHTSIAS · 2020. 4. 7. · Insta 75 Days Revision Plan for UPSC Civil Services Prelims – 2020 This document is the compilation of 100 questions

INSTA 75 Days REVISION PLAN for Prelims 2020 - InstaTests

www.insightsonindia.com 86 Insights IAS

Solution: C

Section 144 of the Criminal Procedure Code (Cr.PC):

• It gives power to a District Magistrate, a sub- divisional Magistrate or any other

Executive Magistrate on behalf of the State Government to issue an order to

an individual or the general public in a particular place or area to “abstain from

a certain act” or “to take certain order with respect to certain property in his

possession or under his management”.

• This order can be passed against a particular individual or general public.

The order can be passed even ex-parte.

• As held by the Supreme Court, mere apprehension of danger is not a

sufficient ground to curb citizens’ rights by invoking Section 144 CrPC.

• Section 144 also empowers the authorities to block the internet access. In

the Anuradha Bhasin case (Kashmir Internet case- Bhasin Doctrine) and

many other cases, the court allowed an internet ban with constant examination

and review.

• Even Social Media activity can be monitored, or it can even be restricted.

Communication blockage can also be imposed, if it will strongly affect public

order.

Duration of Section 144 order:

• No order under Section 144 shall remain in force for more than two months

but the state government can extent the validity for two months and maximum

up to six months. It can be withdrawn at any point of time if situation becomes

normal.

85. Consider the following statements regarding Board of Financial Supervision

(BFS)

1. The Reserve Bank of India performs the supervisory function under the

guidance of the Board for Financial Supervision

2. It provides technical assistance for cooperative banks

3. The primary objective of BFS is to undertake consolidated supervision

of the financial sector

Which of the statements given above is/are not correct?

(a) 1 only

(b) 3 only

(c) 1 and 2 only

(d) None

Solution: D

Page 89: SIMPLYFYING IAS EXAM PREPARATION - INSIGHTSIAS · 2020. 4. 7. · Insta 75 Days Revision Plan for UPSC Civil Services Prelims – 2020 This document is the compilation of 100 questions

INSTA 75 Days REVISION PLAN for Prelims 2020 - InstaTests

www.insightsonindia.com 87 Insights IAS

The Reserve Bank of India performs the supervisory function under the guidance of

the Board for Financial Supervision (BFS). The Board was constituted in November

1994 as a committee of the Central Board of Directors of the Reserve Bank of India

under the Reserve Bank of India (Board for Financial Supervision) Regulations,

1994.

Objective

• The primary objective of BFS is to undertake consolidated supervision of the

financial sector comprising Scheduled Commercial and Co-operative

Banks, All India Financial Institutions, Local Area Banks, Small Finance Banks,

Payments Banks, Credit Information Companies, Non-Banking Finance

Companies and Primary Dealers.

Constitution

• The Board is constituted by co-opting four Directors from the Central Board as

Members and is chaired by the Governor. The Deputy Governors of the

Reserve Bank are ex-officio members. One Deputy Governor, traditionally, the

Deputy Governor in charge of supervision, is nominated as the Vice-Chairman

of the Board.

• In April 2018, a Sub-committee of the Board for Financial Supervision was

constituted, under Para 11 & 12 of the Reserve Bank of India (Board for

Financial Supervision) Regulations, 1994. The Sub-committee performs the

functions and exercises the powers of supervision and inspection under the

Reserve Bank of India Act, 1934 and the Banking Regulation Act, 1949, in

relation to Payments Banks, Small Finance Banks, Local Area Banks, small

Foreign Banks, select scheduled Urban Co-operative Banks, select Non-

Banking Financial Companies and Credit Information Companies.

• The Sub-committee is chaired by the Deputy Governor in charge of supervision

and includes the three Deputy Governors and two Directors of the Central

Board as Members.

BFS Meetings

• The Board is required to meet normally once every month. It deliberates on

inspection reports, periodic reviews related to banking and non-banking sectors

and policy matters arising out of or having relevance to the supervisory

functions of the Reserve Bank.

• The BFS oversees the functioning of Department of Banking Supervision

(DBS), Department of Non-Banking Supervision (DNBS) and Department

of Co-operative Bank Supervision (DCBS) and gives directions on

regulatory and supervisory issues.

Functions

Some of the initiatives taken by the BFS include:

• Fine-tuning the supervisory processes adopted by the Bank for regulated

entities;

Page 90: SIMPLYFYING IAS EXAM PREPARATION - INSIGHTSIAS · 2020. 4. 7. · Insta 75 Days Revision Plan for UPSC Civil Services Prelims – 2020 This document is the compilation of 100 questions

INSTA 75 Days REVISION PLAN for Prelims 2020 - InstaTests

www.insightsonindia.com 88 Insights IAS

• Introduction of off-site surveillance system to complement the on-site

supervision of regulated entities;

• Strengthening the statutory audit processes of banks and enlarging the role of

auditors in the supervisory process;

• Strengthening the internal defences within supervised institutions such as

corporate governance, internal control and audit functions, management

information and risk control systems, review of housekeeping in banks;

• Introduction of supervisory rating system for banks and financial institutions;

• Supervision of overseas operations of Indian banks, consolidated supervision

of banks;

• Technical assistance programme for cooperative banks;

• Introduction of scheme of Prompt Corrective Action Framework for weak banks;

• Guidance regarding fraud risk management framework in banks;

• Introduction of risk based supervision of banks;

• Introduction of an enforcement framework in respect of banks;

• Establishment of a credit registry in respect of large borrowers of supervised

institutions; and

• Setting up a subsidiary of RBI to take care of the IT requirements, including the

cyber security needs of the Reserve Bank and its regulated entities, etc.

https://www.rbi.org.in/Scripts/AboutusDisplay.aspx#FS

86. Which of the following act exempted the Governor-General and the Council

from the jurisdiction of the Supreme Court for the acts done by them in their

official capacity?

(a) Regulating Act of 1773

(b) Amending Act of 1781

(c) Pitt’s India Act of 1784

(d) Charter Act of 1793

Solution: B

Amending Act of 1781

• In a bid to rectify the defects of the Regulating Act of 1773, the British

Parliament passed the Amending Act of 1781, also known as the Act of

Settlement.

The features of this Act were as follows:

• It exempted the Governor-General and the Council from the jurisdiction of the

Supreme Court for the acts done by them in their official capacity. Similarly, it

also exempted the servants of the company from the jurisdiction of the Supreme

Court for their official actions.

Page 91: SIMPLYFYING IAS EXAM PREPARATION - INSIGHTSIAS · 2020. 4. 7. · Insta 75 Days Revision Plan for UPSC Civil Services Prelims – 2020 This document is the compilation of 100 questions

INSTA 75 Days REVISION PLAN for Prelims 2020 - InstaTests

www.insightsonindia.com 89 Insights IAS

• It excluded the revenue matters and the matters arising in the collection of

revenue from the jurisdiction of the Supreme Court.

• It provided that the Supreme Court was to have jurisdiction over all the

inhabitants of Calcutta. It also required the court to administer the personal law

of the defendants i.e., Hindus were to be tried according to the Hindu law and

Muslims were to be tried according to the Mohammedan law.

• It laid down that the appeals from the Provincial Courts could be taken to the

Governor-General-in-Council and not to the Supreme Court.

• It empowered the Governor-General-in Council to frame regulations for the

Provincial Courts and Councils.

87. Consider the following pairs regarding the Committees of the Constituent

Assembly with their chairperson

1. Union Powers Committee – Jawaharlal Nehru

2. Union Constitution Committee – Dr. B.R. Ambedkar

3. States Committee (Committee for Negotiating with States) – Sardar

Patel

4. Steering Committee – Dr. Rajendra Prasad

Which of the pairs given above is/are correctly matched?

(a) 1 and 4 only

(b) 2 and 3 only

(c) 1, 3 and 4 only

(d) 1, 2, 3 and 4

Solution: A

The Constituent Assembly appointed a number of committees to deal with different

tasks of constitution-making. Out of these, eight were major committees and the others

were minor committees. The names of these committees and their chairmen are given

below:

Major Committees

1. Union Powers Committee – Jawaharlal Nehru

2. Union Constitution Committee – Jawaharlal Nehru

3. Provincial Constitution Committee – Sardar Patel

4. Drafting Committee – Dr. B.R. Ambedkar

5. Advisory Committee on Fundamental Rights, Minorities and Tribal and

Excluded Areas – Sardar Patel.

This committee had the following five sub-committees:

1) Fundamental Rights Sub-Committee – J.B. Kripalani

2) Minorities Sub-Committee – H.C. Mukherjee

Page 92: SIMPLYFYING IAS EXAM PREPARATION - INSIGHTSIAS · 2020. 4. 7. · Insta 75 Days Revision Plan for UPSC Civil Services Prelims – 2020 This document is the compilation of 100 questions

INSTA 75 Days REVISION PLAN for Prelims 2020 - InstaTests

www.insightsonindia.com 90 Insights IAS

3) North-East Frontier Tribal Areas and Assam Excluded & Partially

Excluded Areas Sub-Committee – Gopinath Bardoloi

4) Excluded and Partially Excluded Areas (Other than those in Assam)

Sub-Committee – A.V. Thakkar

5) North-West Frontier Tribal Areas Sub-Committee

6. Rules of Procedure Committee – Dr. Rajendra Prasad

7. States Committee (Committee for Negotiating with States) – Jawaharlal Nehru

8. Steering Committee – Dr. Rajendra Prasad

Minor Committees

1. Finance and Staff Committee – Dr. Rajendra Prasad

2. Credentials Committee – Alladi Krishnaswami Ayyar

3. House Committee – B. Pattabhi Sitaramayya

4. Order of Business Committee – Dr. K.M. Munshi

5. Ad-hoc Committee on the National Flag – Dr. Rajendra Prasad

6. Committee on the Functions of the Constituent Assembly – G.V. Mavalankar

7. Ad-hoc Committee on the Supreme Court – S. Varadachari (Not an Assembly

Member)

8. Committee on Chief Commissioners’ Provinces – B. Pattabhi Sitaramayya

9. Expert Committee on the Financial Provisions of the Union Constitution – Nalini

Ranjan Sarkar (Not an Assembly Member)

10. Linguistic Provinces Commission – S.K. Dar (Not an Assembly Member)

11. Special Committee to Examine the Draft Constitution – Jawaharlal Nehru

12. Press Gallery Committee – Usha Nath Sen

13. Ad-hoc Committee on Citizenship – S. Varadachari

88. Consider the following statements regarding Spirulina

1. It is a processed nutribar developed in laboratory atmosphere.

2. It is a source of antioxidants and protect against oxidative damage.

Which of the statements given above is/are correct?

(a) 1 only

(b) 2 only

(c) Both 1 and 2

(d) Neither 1 nor 2

Solution: B

Spirulina is an organism that grows in both fresh and salt water.

• It is a type of cyanobacteria, whi’[\ch is a family of single-celled microbes that

are often referred to as blue-green algae.

Page 93: SIMPLYFYING IAS EXAM PREPARATION - INSIGHTSIAS · 2020. 4. 7. · Insta 75 Days Revision Plan for UPSC Civil Services Prelims – 2020 This document is the compilation of 100 questions

INSTA 75 Days REVISION PLAN for Prelims 2020 - InstaTests

www.insightsonindia.com 91 Insights IAS

• Just like plants, cyanobacteria can produce energy from sunlight via a process

called photosynthesis.

• Spirulina was consumed by the ancient Aztecs but became popular again when

NASA proposed that it could be grown in space for use by astronauts

• A standard daily dose of spirulina is 1–3 grams, but doses of up to 10 grams

per day have been used effectively.

This tiny alga is packed with nutrients. Namely

1) Protein: 4 grams

2) Vitamin B1 (thiamine):

3) Vitamin B2 (riboflavin):

4) Vitamin B3 (niacin):

5) Copper:

6) Iron:

7) It also contains decent amounts of magnesium, potassium and manganese and

small amounts of almost every other nutrient that you need.

Spirulina is a fantastic source of antioxidants, which can protect against oxidative

damage.

Its main active component is called phycocyanin. This antioxidant substance also

gives spirulina its unique blue-green color.

Phycocyanin can fight free radicals and inhibit production of inflammatory signaling

molecules, providing impressive antioxidant and anti-inflammatory effects.

https://www.healthline.com/nutrition/10-proven-benefits-of-spirulina#section2

https://www.thehindu.com/news/cities/Vijayawada/spirulina-to-fight-malnutrition-in-

kids/article19738184.ece

89. Which of the following is/are the federal features of Indian Constitution:

1. Division of powers

2. Supremacy of Constitution

3. Integrated judiciary

4. Bicameralism

Select the correct answer using the code given below:

(a) 1, 2 and 3 only

(b) 2, 3 and 4 only

(c) 1, 2 and 4 only

(d) 1, 2, 3 and 4

Solution: C

Page 94: SIMPLYFYING IAS EXAM PREPARATION - INSIGHTSIAS · 2020. 4. 7. · Insta 75 Days Revision Plan for UPSC Civil Services Prelims – 2020 This document is the compilation of 100 questions

INSTA 75 Days REVISION PLAN for Prelims 2020 - InstaTests

www.insightsonindia.com 92 Insights IAS

The Constitution of India establishes a federal system of Government. It contains all

the usual features of a federation, viz., two Government, division of powers, written

Constitution, supremacy of Constitution, rigidity of Constitution, independent judiciary

and bicameralism.

However, the Indian Constitution also contains a large number of unitary or non-

federal features, viz., a strong Centre, single Constitution, single citizenship, flexibility

of Constitution, integrated judiciary, appointment of state governor by the Centre, all-

India services, emergency provisions and so on.

Extra Learning:

The Constitution of India provides for a federal system of government in the country.

The framers adopted the federal system due to two main reasons : the large size of

the country and its socio-cultural diversity. They realised that the federal system not

only ensures the efficient governance of the country but also reconciles national unity

with regional autonomy.

However, the term ‘federation’ has nowhere been used in the Constitution. Instead,

Article 1 of the Constitution describes India as a ‘Union of States’.

According to Dr B R Ambedkar, the phrase ‘Union of States’ has been preferred to

‘Federation of States’ to indicate two things:

(i) the Indian federation is not the result of an agreement among the states like

the American federation; and

(ii) the states have no right to secede from the federation. The federation is union

because it is indestructible.

The Indian federal system is based on the ‘Canadian model’ and not on the ‘American

model’. The ‘Canadian model’ differs fundamentally from the ‘American model’ in so

far as it establishes a very strong centre.

The Indian federation resembles the Canadian federation

(iii) in its formation (i.e., by way of disintegration);

(iv) in its preference to the term ‘Union’ (the Canadian federation is also called a

‘Union’); and

(v) in its centralising tendency (i.e., vesting more powers in the centre vis-a-vis

the states).

90. Consider the following statements regarding concept of secularism

1. The Western concept of secularism connotes a complete separation

between the religion and the state.

2. The Indian Constitution embodies the negative concept of secularism.

Which of the statements given above is/are correct?

Page 95: SIMPLYFYING IAS EXAM PREPARATION - INSIGHTSIAS · 2020. 4. 7. · Insta 75 Days Revision Plan for UPSC Civil Services Prelims – 2020 This document is the compilation of 100 questions

INSTA 75 Days REVISION PLAN for Prelims 2020 - InstaTests

www.insightsonindia.com 93 Insights IAS

(a) 1 only

(b) 2 only

(c) Both 1 and 2

(d) Neither 1 nor 2

Solution: A

The Western concept of secularism connotes a complete separation between

the religion (the church) and the state (the politics). This negative concept of

secularism is inapplicable in the Indian situation where the society is multi-religious.

Hence, the Indian Constitution embodies the positive concept of secularism, i.e.,

giving equal respect to all religions or protecting all religions equally. The

Western concept of secularism connotes a complete separation between the religion

(the church) and the state (the politics). This negative concept of secularism is

inapplicable in the Indian situation where the society is multi-religious. Hence, the

Indian Constitution embodies the positive concept of secularism, i.e., giving equal

respect to all religions or protecting all religions equally.

91. Consider the following statements regarding formation of states in India

1. Article 3 empowers the Parliament to ‘admit into the Union of India, or

establish, new states on such terms and conditions as it thinks fit’.

2. President has to refer the bill to the state legislature concerned for

expressing its views within a specified period and he is bound by the

views of the state legislature.

Which of the statements given above is/are correct?

(a) 1 only

(b) 2 only

(c) Both 1 and 2

(d) Neither 1 nor 2

Solution: D

Article 3 authorizes the Parliament to:

(a) form a new state by separation of territory from any state or by uniting two or more

states or parts of states or by uniting any territory to a part of any state;

(b) increase the area of any state;

(c) diminish the area of any state;

(d) alter the boundaries of any state; and

(e) alter the name of any state.

Page 96: SIMPLYFYING IAS EXAM PREPARATION - INSIGHTSIAS · 2020. 4. 7. · Insta 75 Days Revision Plan for UPSC Civil Services Prelims – 2020 This document is the compilation of 100 questions

INSTA 75 Days REVISION PLAN for Prelims 2020 - InstaTests

www.insightsonindia.com 94 Insights IAS

However, Article 3 lays down two conditions in this regard: one, a bill contemplating

the above changes can be introduced in the Parliament only with the prior

recommendation of the President; and two, before recommending the bill, the

President has to refer the same to the state legislature concerned for expressing

its views within a specified period.

Article 2 empowers the Parliament to ‘admit into the Union of India, or establish, new

states on such terms and conditions as it thinks fit’.

Notably, Article 2 relates to the admission or establishment of new states that are not

part of the Union of India. Article 3, on the other hand, relates to the formation of or

changes in the existing states of the Union of India. In other words, Article 3 deals with

the internal re-adjustment inter se of the territories of the constituent states of the

Union of India.

The President (or Parliament) is not bound by the views of the state legislature and

may either accept or reject them, even if the views are received in time.

92. Consider the following statements regarding Sovereign Gold Bond Scheme

1. Bonds can be used as collateral for loans.

2. The minimum permissible investment limit will be 1 gram of gold for

individual.

Which of the statements given above is/are correct?

(a) 1 only

(b) 2 only

(c) Both 1 and 2

(d) Neither 1 nor 2

Solution: C

Government of India, in consultation with the Reserve Bank of India, has decided to

issue Sovereign Gold Bonds. The Bonds will be sold through Scheduled

Commercial banks (except Small Finance Banks and Payment Banks), Stock

Holding Corporation of India Limited (SHCIL), designated post offices, and

recognized stock exchanges viz., National Stock Exchange of India Limited and

Bombay Stock Exchange Limited.

About the Sovereign Gold Bond Scheme:

• The sovereign gold bond was introduced by the Government in 2015.

• Government introduced these bonds to help reduce India’s over dependence

on gold imports.

• The move was also aimed at changing the habits of Indians from saving in

physical form of gold to a paper form with Sovereign backing.

Page 97: SIMPLYFYING IAS EXAM PREPARATION - INSIGHTSIAS · 2020. 4. 7. · Insta 75 Days Revision Plan for UPSC Civil Services Prelims – 2020 This document is the compilation of 100 questions

INSTA 75 Days REVISION PLAN for Prelims 2020 - InstaTests

www.insightsonindia.com 95 Insights IAS

Key facts:

• Eligibility: The bonds will be restricted for sale to resident Indian entities,

including individuals, HUFs, trusts, universities and charitable institutions.

• Denomination and tenor: The bonds will be denominated in multiples of

gram(s) of gold with a basic unit of 1 gram. The tenor will be for a period of 8

years with exit option from the 5th year to be exercised on the interest payment

dates.

• Minimum and Maximum limit: The minimum permissible investment limit will

be 1 gram of gold, while the maximum limit will be 4 kg for individual, 4 kg for

HUF and 20 kg for trusts and similar entities per fiscal (April-March) notified by

the government from time to time.

• Joint Holder: In case of joint holding, the investment limit of 4 kg will be applied

to the first applicant only.

• Collateral: Bonds can be used as collateral for loans. The loan-to-value (LTV)

ratio is to be set equal to ordinary gold loan mandated by the Reserve Bank

from time to time.

• Tenor: The tenor of the Bond will be for a period of 8 years with exit option after

5th year to be exercised on the interest payment dates.

• Interest rate: The investors will be compensated at a fixed rate of 2.50 percent

per annum payable semi-annually on the nominal value.

https://www.insightsonindia.com/2019/10/01/sovereign-gold-bond-scheme/

93. Which of the following fundamental/constitutional rights is/are available only to

citizens of India

1. Right against discrimination under Article 15

2. Right to equality of opportunity in the matter of public employment

3. Right to freedom of speech and expression

4. Right to vote in elections to the Lok Sabha and state legislative assembly

Select the correct answer using the code given below:

(a) 1, 2 and 3 only

(b) 2, 3 and 4 only

(c) 1, 2 and 4 only

(d) 1, 2, 3 and 4

Solution: D

Like any other modern state, India has two kinds of people—citizens and aliens.

Citizens are full members of the Indian State and owe allegiance to it. They enjoy all

civil and political rights. Aliens, on the other hand, are the citizens of some other state

and hence, do not enjoy all the civil and political rights.

Page 98: SIMPLYFYING IAS EXAM PREPARATION - INSIGHTSIAS · 2020. 4. 7. · Insta 75 Days Revision Plan for UPSC Civil Services Prelims – 2020 This document is the compilation of 100 questions

INSTA 75 Days REVISION PLAN for Prelims 2020 - InstaTests

www.insightsonindia.com 96 Insights IAS

They are of two categories—friendly aliens or enemy aliens.

• Friendly aliens are the subjects of those countries that have cordial relations

with India. Enemy aliens, on the other hand, are the subjects of that country

that is at war with India. They enjoy lesser rights than the friendly aliens, eg,

they do not enjoy protection against arrest and detention (Article 22).

The Constitution confers the following rights and privileges on the citizens of India (and

denies the same to aliens):

1. Right against discrimination on grounds of religion, race, caste, sex or place of

birth (Article 15).

2. Right to equality of opportunity in the matter of public employment (Article 16).

3. Right to freedom of speech and expression, assembly, association, movement,

residence and profession (Article 19).

4. Cultural and educational rights (Articles 29 and 30).

5. Right to vote in elections to the Lok Sabha and state legislative assembly.

6. Right to contest for the membership of the Parliament and the state legislature.

Eligibility to hold certain public offices, that is, President of India, Vice-President of

India, judges of the Supreme Court and the high courts, Governor of states, Attorney

General of India and Advocate General of states.

94. Which of the following elements of Rule of Law are applicable to the Indian

System

1. Absence of arbitrary power

2. Equality before the law

3. The primacy of the rights of the individual

Select the correct answer using the code given below:

(a) 1 and 2 only

(b) 2 and 3 only

(c) 1 and 3 only

(d) 1, 2 and 3

Solution: A

The concept of ‘equality before law’ is an element of the concept of ‘Rule of Law’,

propounded by A.V. Dicey, the British jurist.

His concept has the following three elements or aspects:

(i) Absence of arbitrary power, that is, no man can be punished except for a

breach of law.

Page 99: SIMPLYFYING IAS EXAM PREPARATION - INSIGHTSIAS · 2020. 4. 7. · Insta 75 Days Revision Plan for UPSC Civil Services Prelims – 2020 This document is the compilation of 100 questions

INSTA 75 Days REVISION PLAN for Prelims 2020 - InstaTests

www.insightsonindia.com 97 Insights IAS

(ii) Equality before the law, that is, equal subjection of all citizens (rich or poor,

high or low, official or non-official) to the ordinary law of the land administered

by the ordinary law courts.

(iii) The primacy of the rights of the individual, that is, the constitution is the

result of the rights of the individual as defined and enforced by the courts of

law rather than the constitution being the source of the individual rights.

The first and the second elements are applicable to the Indian System and not the

third one. In the Indian System, the constitution is the source of the individual rights.

The Supreme Court held that the ‘Rule of Law’ as embodied in Article 14 is a ‘basic

feature’ of the constitution. Hence, it cannot be destroyed even by an amendment.

95. The School Education Quality Index has been launched by

(a) Pratham NGO

(b) Institute of Human Resources Development (IHRD)

(c) Ministry of Human Resource Development

(d) NITI Aayog

Solution: D

NITI Aayog releases the first edition of School Education Quality Index (SEQI).

About the School Education Quality Index (SEQI):

• Developed by NITI Aayog to evaluate the performance of States and Union

Territories (UTs) in the school education sector.

• Aim: To bring an ‘outcomes’ focus to education policy by providing States and

UTs with a platform to identify their strengths and weaknesses and undertake

requisite course corrections or policy interventions.

• The index is developed through a collaborative process, including key

stakeholders such as Ministry of Human Resource and Development (MHRD),

the World Bank and sector experts.

It consists of 30 critical indicators that assess the delivery of quality education. These

indicators are categorized as below:

Category 1: Outcomes;

• Domain 1: Learning outcomes

• Domain 2: Access outcomes

• Domain 3: Infrastructure and facilities for outcomes

• Domain 4: Equity outcomes

Category 2: Governance processes aiding outcomes.

Page 100: SIMPLYFYING IAS EXAM PREPARATION - INSIGHTSIAS · 2020. 4. 7. · Insta 75 Days Revision Plan for UPSC Civil Services Prelims – 2020 This document is the compilation of 100 questions

INSTA 75 Days REVISION PLAN for Prelims 2020 - InstaTests

www.insightsonindia.com 98 Insights IAS

Significance of the index:

• Schooling should result in successful learning outcomes. In this regard, SEQI

acts as a credible system of assessment and helps to design necessary

remedial actions.

How are they ranked?

• To facilitate a like-for-like comparison, States and UTs have been grouped as

Large States, Small States and UTs.

• States’ and UTs’ performance on Learning Outcomes is driven by their results

on the National Achievement Survey (NAS) 2017.

• Their performance on Access Outcomes is primarily driven by enrolment ratios

at the secondary level and transition rates from upper-primary to secondary

level.

• In terms of Infrastructure & Facilities for Outcomes, States’ and UTs’

performance is strongly linked to the presence of Computer Aided-Learning

(CAL) at the elementary level and vocational education at the secondary and

senior-secondary level.

Performance of various states:

• Kerala has emerged on top among 20 large states in terms of quality of school

education, followed by Rajasthan and Karnataka, while the most-populous

Uttar Pradesh was ranked at the bottom position during 2016-17.

• Among 20 large states in the country 18 have improved their overall

performance between 2015-2016 and 2016-2017, which is otherwise referred

to as the incremental performance in the report.

• The highest incremental performance has been recorded in Kerala. As against

77.6% score in 2015-2016, the state has recorded 82.2% in 2016-2017.

• The overall performance has declined in Karnataka and Uttarakhand.

• All seven union territories have shown an improvement in their overall

performance scores.

• The performance and ranks have decreased in Mizoram, Sikkim and Arunachal

Pradesh.

https://www.insightsonindia.com/2019/10/01/school-education-quality-index-seqi/

96. Consider the following statements regarding National Pension System (NPS)

1. It was launched in 2004 for government employees.

2. Any Indian citizen, resident or non-resident and OCIs are eligible to join

NPS till the age of 65 years.

3. The scheme is managed by National Council for Senior Citizens.

Which of the statements given above is/are correct?

(a) 2 only

(b) 1 and 3 only

Page 101: SIMPLYFYING IAS EXAM PREPARATION - INSIGHTSIAS · 2020. 4. 7. · Insta 75 Days Revision Plan for UPSC Civil Services Prelims – 2020 This document is the compilation of 100 questions

INSTA 75 Days REVISION PLAN for Prelims 2020 - InstaTests

www.insightsonindia.com 99 Insights IAS

(c) 1 and 2 only

(d) 1, 2 and 3

Solution: C

Pension Fund Regulatory and Development Authority (PFRDA) has now

permitted Overseas Citizen of India (OCI) to enroll in National Pension Scheme (NPS)

at par with Non-Resident Indians.

What is National Pension System (NPS)?

• It is a government-sponsored pension scheme. It was launched in January

2004 for government employees. However, in 2009, it was opened to all

sections.

• The scheme allows subscribers to contribute regularly in a pension account

during their working life. On retirement, subscribers can withdraw a part of the

corpus in a lumpsum and use the remaining corpus to buy an annuity to secure

a regular income after retirement.

• This system is managed by PFRDA (Pension Fund Regulatory and

Development Authority).

Who can join NPS?

• Any Indian citizen between 18 and 65 years can join NPS.

• An NRI can join NPS. However, the account will be closed if there is a change

in the citizenship status of the NRI.

• Now, any Indian citizen, resident or non-resident and OCIs are eligible to join

NPS till the age of 65 years.

https://www.insightsonindia.com/2019/10/31/national-pension-scheme-2/

97. Consider the following statements regarding Directive Principles

1. The Directive Principles resemble the ‘Instrument of Instructions’

enumerated in the Government of India Act of 1935.

2. The Directive Principles are non-justiciable in nature.

3. The Directive Principles help the courts in examining and determining

the constitutional validity of a law.

Which of the statements given above is/are correct?

(a) 1 and 2 only

(b) 2 and 3 only

(c) 1 and 3 only

(d) 1, 2 and 3

Page 102: SIMPLYFYING IAS EXAM PREPARATION - INSIGHTSIAS · 2020. 4. 7. · Insta 75 Days Revision Plan for UPSC Civil Services Prelims – 2020 This document is the compilation of 100 questions

INSTA 75 Days REVISION PLAN for Prelims 2020 - InstaTests

www.insightsonindia.com 100 Insights IAS

Solution: D

• The Directive Principles resemble the ‘Instrument of Instructions’

enumerated in the Government of India Act of 1935. In the words of Dr. B.R.

Ambedkar, ‘the Directive Principles are like the instrument of instructions, which

were issued to the Governor-General and to the Governors of the colonies of

India by the British Government under the Government of India Act of 1935.

• The Directive Principles are non-justiciable in nature, that is, they are not

legally enforceable by the courts for their violation.

• The Directive Principles, though non-justiciable in nature, help the courts in

examining and determining the constitutional validity of a law. The Supreme

Court has ruled many a times that in determining the constitutionality of any

law, if a court finds that the law in question seeks to give effect to a Directive

Principle, it may consider such law to be ‘reasonable’ in relation to Article 14

(equality before law) or Article 19 (six freedoms) and thus save such law from

unconstitutionality.

Extra Learning:

Difference between Fundamental Rights and Directive Principles:

Page 103: SIMPLYFYING IAS EXAM PREPARATION - INSIGHTSIAS · 2020. 4. 7. · Insta 75 Days Revision Plan for UPSC Civil Services Prelims – 2020 This document is the compilation of 100 questions

INSTA 75 Days REVISION PLAN for Prelims 2020 - InstaTests

www.insightsonindia.com 101 Insights IAS

98. Consider the following statements regarding Pardoning powers of President.

1. The power of pardon shall be exercised by the President on the advice

of Council of Ministers.

2. Governor can grant pardon to the sentence of death.

3. The constitution provides the mechanism to question the legality of

decisions of President or governors exercising mercy jurisdiction.

Which of the statements given above is/are correct?

(a) 1 only

(b) 2 and 3 only

(c) 1 and 2 only

(d) 1, 2 and 3

Solution: A

The President has commuted death sentences to life imprisonment in at least 20 cases

over the past nine years, based on the recommendations received from the Ministry

of Home Affairs (MHA).

Clemency powers of the President under article 72:

It says that the President shall have the power to grant pardons, reprieves, respites or

remissions of punishment or to suspend, remit or commute the sentence of any person

convicted of any offence.

• Pardon –A pardon completely absolves the offender from all sentences and

punishment and disqualifications and places him in the same position as if he

had never committed the offence.

• Commutation– Commutation means exchange of one thing for another. In

simple words to replace the punishment with less severe punishment. For

example for Rigorous imprisonment-simple imprisonment.

• Reprieve– Reprieve means temporary suspension of death sentence. For

example- pending a proceeding for pardon or commutation.

• Respite – Respite means awarding a lesser punishment on some special

grounds. For example- the Pregnancy of women offender.

• Remissions– Remission means the reduction of the amount of sentence

without changing its character, for example, a sentence of 1 year may be

remitted to 6 months.

The President can exercise these powers:

• In all cases where the punishment or sentence is by a court martial;

• In all cases where the punishment or sentence is for an offence against any law

relating to a matter to which the executive power of the Union extends;

• In all cases where the sentence is a sentence of death.

Page 104: SIMPLYFYING IAS EXAM PREPARATION - INSIGHTSIAS · 2020. 4. 7. · Insta 75 Days Revision Plan for UPSC Civil Services Prelims – 2020 This document is the compilation of 100 questions

INSTA 75 Days REVISION PLAN for Prelims 2020 - InstaTests

www.insightsonindia.com 102 Insights IAS

The pardoning power of President is wider than the governor and it differs in the

following two ways:

1. The power of the President to grant pardon extends in cases where the

punishment or sentence is by a Court Martial but Article 161 does not

provide any such power to the Governor.

2. The President can grant pardon in all cases where the sentence given is

sentence of death but pardoning power of Governor does not extend to

death sentence cases.

Key facts:

• This power of pardon shall be exercised by the President on the advice of

Council of Ministers.

• Further, the constitution does not provide for any mechanism to question the

legality of decisions of President or governors exercising mercy jurisdiction.

• But the SC in Epuru Sudhakar case has given a small window for judicial

review of the pardon powers of President and governors for the purpose of

ruling out any arbitrariness.

• The court has earlier held that court has retained the power of judicial review

even on a matter which has been vested by the Constitution solely in the

Executive.

https://www.insightsonindia.com/2019/10/05/pardoning-powers-of-president/

99. Which of the following provisions of the Indian Constitution is/are amended by

Special Majority of Parliament and Consent of States

1. Goods and Services Tax Council

2. Representation of states in Parliament

3. Fundamental Rights and Directive Principles of State Policy

Select the correct answer using the code given below:

(a) 1 and 2 only

(b) 2 and 3 only

(c) 1 and 3 only

(d) 1, 2 and 3

Solution: A

Article 368 provides for two types of amendments, that is, by a special majority of

Parliament and also through the ratification of half of the states by a simple

majority. But, some other articles provide for the amendment of certain provisions of

the Constitution by a simple majority of Parliament, that is, a majority of the

members of each House present and voting (similar to the ordinary legislative

Page 105: SIMPLYFYING IAS EXAM PREPARATION - INSIGHTSIAS · 2020. 4. 7. · Insta 75 Days Revision Plan for UPSC Civil Services Prelims – 2020 This document is the compilation of 100 questions

INSTA 75 Days REVISION PLAN for Prelims 2020 - InstaTests

www.insightsonindia.com 103 Insights IAS

process). Notably, these amendments are not deemed to be amendments of the

Constitution for the purposes of Article 368.

Therefore, the Constitution can be amended in three ways:

(a) Amendment by simple majority of the Parliament,

(b) Amendment by special majority of the Parliament, and

(c) Amendment by special majority of the Parliament and the ratification of half of the

state legislatures.

By Special Majority of Parliament and Consent of States

1. Election of the President and its manner.

2. Extent of the executive power of the Union and the states.

3. Supreme Court and high courts.

4. Distribution of legislative powers between the Union and the states.

5. Goods and Services Tax Council.

6. Any of the lists in the Seventh Schedule.

7. Representation of states in Parliament.

8. Power of Parliament to amend the Constitution and its procedure (Article 368

itself).

Extra Learning:

By Simple Majority of Parliament

A number of provisions in the Constitution can be amended by a simple majority of the

two Houses of Parliament outside the scope of Article 368.

These provisions include:

1. Admission or establishment of new states.

2. Formation of new states and alteration of areas, boundaries or names of existing

states.

3. Abolition or creation of legislative councils in states.

4. Second Schedule—emoluments, allowances, privileges and so on of the

president, the governors, the Speakers, judges, etc.

5. Quorum in Parliament.

6. Salaries and allowances of the members of Parliament.

7. Rules of procedure in Parliament.

8. Privileges of the Parliament, its members and its committees.

9. Use of English language in Parliament.

10. Number of puisne judges in the Supreme Court.

11. Conferment of more jurisdiction on the Supreme Court.

12. Use of official language.

13. Citizenship—acquisition and termination.

14. Elections to Parliament and state legislatures.

15. Delimitation of constituencies.

16. Union territories.

Page 106: SIMPLYFYING IAS EXAM PREPARATION - INSIGHTSIAS · 2020. 4. 7. · Insta 75 Days Revision Plan for UPSC Civil Services Prelims – 2020 This document is the compilation of 100 questions

INSTA 75 Days REVISION PLAN for Prelims 2020 - InstaTests

www.insightsonindia.com 104 Insights IAS

17. Fifth Schedule—administration of scheduled areas and scheduled tribes.

18. Sixth Schedule—administration of tribal areas.

By Special Majority of Parliament

The provisions which can be amended by this way includes:

(i) Fundamental Rights;

(ii) Directive Principles of State Policy; and

(iii) All other provisions which are not covered by the first and third categories.

100. Which among the following states has launched Kanya Sumangala Yojana?

(a) Haryana

(b) West Bengal

(c) Uttar Pradesh

(d) Kerala

Solution: C

Kanya Sumangala Yojana to be launched for girl child in UP.

Key features:

• It will provide a fund worth Rs 15000 to every family where a girl child is born.

The amount will be released to the family in a phased manner.

• The scheme has been designed in a way that the parents will have to take

proper care of the girl child with respect to her health and education and other

aspects, in order to get the benefit.

https://www.insightsonindia.com/2019/10/25/kanya-sumangala-yojana/

Page 107: SIMPLYFYING IAS EXAM PREPARATION - INSIGHTSIAS · 2020. 4. 7. · Insta 75 Days Revision Plan for UPSC Civil Services Prelims – 2020 This document is the compilation of 100 questions

www.insightsonindia.com INSIGHTS IAS

TESTIMONIALS INSTA 75 Days REVISION PLAN for Prelims 2020 – InstaTests

http://disq.us/p/282plo8

http://disq.us/p/282palg

http://disq.us/p/2839b2u

http://disq.us/p/28595xc

http://disq.us/p/2857k0t

http://disq.us/p/2857hhq

http://disq.us/p/2856dfp

http://disq.us/p/2858me1

http://disq.us/p/28592b1

Page 108: SIMPLYFYING IAS EXAM PREPARATION - INSIGHTSIAS · 2020. 4. 7. · Insta 75 Days Revision Plan for UPSC Civil Services Prelims – 2020 This document is the compilation of 100 questions

www.insightsonindia.com INSIGHTS IAS

http://disq.us/p/2855z70

http://disq.us/p/2855vwp

http://disq.us/p/2855yka

http://disq.us/p/2852u5r

http://disq.us/p/2854jny

http://disq.us/p/285733x

http://disq.us/p/2856vij